You are on page 1of 42

1

CHAPTER

Solutions

Solid Solid Homogeneous mixture of two


INTRODUCTION
9. or more metals (i.e. alloys) e.g.
A solution may be defined as a homogeneous mixture of a single copper In gold. zinc In copper.
phase containing two or more of the chemical species dispersed
on a molecular scale. The characteristics of any one section of CONCENTRATION TERMS
the homogeneous solution will be completely identical to those Before proceeding further, it is worth reviewing the units in which
of any other section of the solution. Depending upon the number the concentration of the solution may be expressed.
of total constituents present in the solution, it is called binary
Concentration Terms
solution (two constituents), ternary solution (three constituents), ™ % Concentration
quaternary solution (four constituents) and so on. weight of solute (g)
(i) Solvent: The liquid or medium of dissolution which allows (i) % w/w = × 100
weight of solution (g)
the solute to dissolve in it so as to form a solution is called
a solvent. Example: 10% w/w urea solution = 10 gm of urea is present
in 100 gm of solution.
(ii) Solute: The substance which dissolves or disappears in the
solvent to form a solution is called solute. (ii) % wt/vol. (w/v)
% w/v = wt. of solute/100 mL of solution
Solute + Solvent = Solution.
weight of solute (g)
(iii) The component which has the same physical state in pure % w/v = × 100
form as the solution is called solvent and the other is called volume of solution (mL)
solute. Example, in case of solution of sugar and water, Example: 10% (w/v) urea solution. = 10 gm of urea is present
sugar is the solute and water is solvent. in 100 mL of solution.
(iv) If both the components have same state as the solution, the volume of solute
one component which is in excess is called solvent and the (iii) % v/v = × 100
volume of solution
other is called solute. Example, alcohol in water, benzene
in toluene etc.
Example: 10% v/v ethanol aqueous solution → 10 mL
of ethanol in 100 mL of solution.
Types of Solution
= 10 mL of ethanol in 90 mL of H2O

S.No. Solvent Solute Examples
™ Strength of solution in g/L
1. Gas Gas Mixture of gases, air.
Weight of solute (in gram) per litre (1000 mL) of solution.
2. Gas Liquid Water vapour in air, mist.
Example: 10% (w/v) sucrose solution, then specify its
Gas Solid Sublimation of a solid into a gas,
3. concen­tration in g/L
smoke storms.
100 mL _____ 10 g
Liquid Gas CO2 gas dissolve in water (aerated
4. 10
drink), soda water.
\ 1000 mL _____ × 1000 = 100 g/L
Liquid Liquid Mixture of miscible liquids e.g. 100
5.
alcohol in water. ™ Molarity (M) = No. of moles of solute per litre of solution.
6. Liquid Solid Salt in water, sugar in water.
n w 1000
Solid Gas Adsorption of gases over metals, Molarity = =   ×
7. V (in L)  M  V (in mL)
hydrogen over palladium.
Solid Liquid Mercury in zinc, mercury in gold No. of moles of solute = molarity × volume ( in L)
8. No. of milli moles of solute = molarity × volume (in mL)
i.e. all amalgams.
Molality (m) = No. of moles of solute per kg (1000 g) of solvent.
™
Let w gram of solute and Molar mass = M (g/mole) is CONVERSION OF
dissolved in ‘W’ gram of solvent. CONCENTRATION TERMS
w 1000 moles × 1000
Molality =   × or molality = 1. Molarity and % solute by mass: Let d = density of
M W(g) W(g)of solvent solution in g/mL and let it contains x% (w/w) solute by
Normality (N) = No. of equivalents per litre of solution
™ x × d ×10
mass. M =
no.of equivalents of solute mA
=
volume of solution (in L)
 No. of equivalents = normality × volume (in L)
™ Specific gravity has no units and its numerical value
(Normality = n-factor × molarity) equals density in g/mL.
Molar mass
 Equivalent mass =
n − factor 2. Molality and mole fraction: Consider a binary solution
Mass of the species consisting of two components A (Solute) and B (Solvent).
 No. of equivalents of a species =
equivalent mass Let XA & XB are the mole fraction of A & B respectively.
™ Formality (F) :
nA nB
 It may be defined as the number of gram formula masses of the XA = , XB =
ionic solutes dissolved per litre of the solution. Mathematically nA + nB nA + nB
Mass of the ionic solute (in gram) If molality of solution be m then:
Formality (F) =
Formula mass of the solute × Volume of solution(L) nA nA
= m × 1000 = × 1000
 Commonly, the term formality is used to express the mass of solvent n B × MB
concentration of the ionic solids which do not exist as where MB is the molecular wt. of the solvent B.
molecule but as network of ions. mole fraction of A 1000
X A 1000 ×
™ Mole Fraction =m × ⇒m=
XB MB mole fraction of B M B
For binary mixture
mole fraction of solute 1000
moles of solute n = m ×
Xsolute = = mole fraction of solvent molecular wt. of solvent
total moles in solution n+N
3. Mole fraction of solute into molarity of solution
moles of solvent N
XSolvent = = X 2 d ×1000
Total moles in solution n+N M=
X1 M1 + M 2 X 2
Xsolute + XSolvent = 1 Mole fraction of solvent and solute are X1 and X2 so X1 + X2 = 1
[Here n ⇒ moles of solute and N ⇒ moles of solvent] Suppose total mole of solution is = 1 then mole of solute and
™ ppm (Parts Per Million) solute and solvent are X2 & X1 respectively
wt.of solute (in g) weight of solute = X2M2, weight of solvent = X1M1
(a) ppm (w/w) = × 106
wt.of solution (in g) & total wt. of solution = X1M1 + X2M2
wt.of solute (in g) X M + X2M2 X M + X2M2
volume of solution = 1 1 ml = 1 1 L
(b) ppm (w/v) = × 106 d d × 1000
vol. of solution (in mL)
moles of solute X 2 × d × 1000
(c) ppm (moles/moles) = × 106 molarity (M) =
moles of solution X 1 M1 + X 2 M 2

Key Note 4. Molarity into mole fraction X2 = 1000M/ [1000d– MM2]


Molarity = M moles solute in 1000 ml of solution
So, moles of solute = M & mass of solution = d × 1000
In general p.p.m is used for very dilute solutions. wt. of solute = MM2 & wt. of solvent = (1000d – MM2)
™ In very dilute solution, mass of solution will be Where M2 is molar mass of solute
approximately taken as equal to mass of solvent. mole fraction of solute = 1000M / [1000d – MM2]
™ Sum of mole fractions of all components is always one. mM1
5. Molality into mole fraction X2 =
W 1000 + mM1
™ Mole fraction, ppm, molality, %   are always Molality = moles of solute in 1000 gm of solvent = m
W
1000
independent of the temperature. moles of solvent = where M1 is molar mass of solvent
M1
™ Normality, molarity, %  V  , %  W  of a solution mole fraction X2 =
m
=
mM1
   
V V 1000 1000 + mM1
+m
changes with change in temperature. M1

P
2 W JEE (XII) Module-1 CHEMISTRY
md ×1000
6. Molality into molarity M =
1000 + mM 2 Identify the correct matching
Molality = m moles of solute in 1000 gm of solvent
(i) (ii) (iii)
mole of solute = m & weight of solute = mM2
Weight of solution = 1000 + mM2 (a) Liquid in solid Atmosphere Gas in liquid
1000 + mM 2
volume of solution = mL = 1000 + mM 2 L (b) Liquid in gas Alloys Liquid in solid
d d × 1000
m × d × 1000 (c) Liquid in liquid Rubber Solid in gas
molarity =
1000 + mM 2 (d) Gas in liquid Rubber Liquid in liquid
M ×1000
7. Molarity into Molality m = Sol. Liquid in liquid – Rubber – Solid in gas
1000d - MM 2
M1 and M2 are molar masses of solvent and solute. Example 3: A storage battery contains a solution of H2SO4
Molarity = M mole of solute in 1000 ml of solution 30% by weight. Find
moles of solute = M & weight of solute = MM2
weight of solution = 1000d (i) Molality (ii) Molarity
mass of solvent = 1000d – MM2 (iii) Normality (iv) Mole fraction of
M × 1000 H2SO4
molality =
1000d − MM 2  (Given density of solution = 1.2 gm/cm3)
1 M  Sol. 30% by weight ⇒ Wsolute = 30 gm, Wsolution = 100 gm,
=
on simplifying d M + 2 
 m 1000  Wsolvent = 70 gm
Dilution & Mixing of two liquids 100
Vsolution = mL
™ Upon dilution no. of moles of solute remains constant. If a 1.2
particular solution having volume V1 mL and molarity M1 is 30 70
diluted upto volume V2 mL. nsolute = = 0.306 nsolvent = = 3.889
98 18
M1V1 = M2V2
M2 : final molarity n solute × 1000 0.306 × 1000
(i) Molality = = = 4.37 m
™ If a solution having volume V1 and molarity M1 is mixed with Wsolvent 70
another solution of same solute having volume V2 & molarity
n solute × 1000 0.306 × 1000
M2 then M1V1 + M2V2 = MR (V1 + V2) (ii) Molarity = = = 3.67 M
Vsolution 100
M V +M 2 V2
MR = Resultant molarity = 1 1 1.2
V1 +V2
(iii) Normality = Wsolute × 1000 = 30 × 1000 = 7.34 N
Esolute × Vsolution 100
49 ×
1.2
Alternatively
Example 1: If we have 6% w/w urea solution with density Normality = n-factor × Molarity
1.060 g/mL, then calculate its strength in g/L.
= 2 × 3.67 = 7.34
Sol. 6 g urea is present in 100 gm solution.
X H 2SO4 0.306
100  mass  =
(iv) X =
6 g in n H 2SO4 + n H 2 O 0.306 + 3.889 = 0.073
mL  density = H SO
 2 4

1.060  volume 
100
mL → 6 gm
1.060
6
∴ 1000 mL = × 1.060 × 1000
100
= 10.6 × 6= 63.6 g/L
1. A sample of H2SO4 (density 1.8 g/mL) is labelled as
Example 2: 74.66% by weight. What is the molarity of acid?
(i) Which type of solution is milk? (a) 15.2 M (b) 16.3 M
(ii) Name a solid - solid type solution.
(iii) Which type of solution is smoke? (c) 17.5 M (d) 13.7 M

Solutions 3
SOLUTIONS OF SOLID/ Heat of hydration: These ions get hydrated. The ions
™
LIQUID IN LIQUID hold the water molecules by ion dipole attraction. Heat
is liberated during the process of hydration. Thus it is an
Solubility of a solid in liquid exothermic process.
(i) Introduction: Different solutes dissolve to a different (a) If lattice energy > hydration energy, the system cools
extent in the same mass of a solvent i.e., they have different down.
solubilities. Solubility is thus the ability of a solute to
dissolve in a particular solvent. Examples: NaNO3, KNO3, KCl, NH4Cl etc,
(ii) Definition: The solubility of a particular solute in a solvent (solubility increases with rise in temperature are).
is the maximum amount of solute that will dissolve in 100 g (b) If lattice energy < hydration energy, the system heats
solvent. up.
Wt. of solute Examples: NaOH, Na2CO3, Na2SO4, all gases, etc,
Solubility = × 100
Wt. of solvent (solubility decreases with rise in temperature are).
(iii) Factors affecting solubility: (c) If lattice energy = hydration energy, no heat
(a) Nature of solute and solvent: Like dissolves like. exchange. In such cases, temperature has little effect
Polar solutes dissolves in polar solvents and non polar on solubility.
solutes dissolve in non polar solvents. NaCl dissolves Example: NaCl, Li2SO4 (solubility fairly increases
in water because NaCl and water both are polar. CS2 or no change with rise in temperature).
dissolves in benzene because CS2 and C6H6 both are
™ Pressure has no effect on solubility for solid- liquid
non polar.
solutions.
(b) Size of solute particles: Dissolution is a surface
phenomenon like the evaporation. So, increase
of surface area of the solute increases the rate of
dissolution. The surface area of a solid solute can be SOLUBILITY OF GASES IN
increased by converting it into powder. The powdered LIQUID SOLUTIONS
solute dissolves more easily than the large crystals,
as in the former case, larger surface area is in contact Henry’s law deals with effect of pressure on the solubility of gas.
with the solvent.
(c) Effect of Temperature: Solubility may increase or Statement
decrease with increase of temperature. This depend The solubility of a gas in a liquid at a given temperature is directly
on the enthalpy change of the solution. If the solute proportional to the partial pressure at which it is dissolved.
dissolves with the evolution of heat, solubility Let X = Mole fraction of gas at a given temperature as a
decreases with increase of temperature. On the other
measure of its solubility.
hand if the solute dissolves with the absorption of heat,
solubility increases with rise in temperature. P = Partial pressure of gas in equilibrium with the
Example: When ammonium chloride or silver nitrate solution.
is dissolved in water, the solution gets cooled. Then according to Henry’s law,
X ∝ P
NH 4 Cl(s) + H 2 O()  → NH 4 Cl(s) − heat
or P ∝ X
For such solutions, solubility increases on increase of
temperature. or P = KHX
Example: When calcium oxide or lithium carbonate where KH = Henry law constant.
is placed in water the solution gets heated.
CaO(s) + H 2 O  → Ca(OH) 2 (aq) + Heat Key Note
For such solutions, solubility decreases on increase of
temperature. (i) If a mixture of gases is brought in contact with solvent,
each constituent gas dissolves in proportion to its
Key Note partial pressure. It means Henry’s law applies to each
gas independent of the presence of other gas.
The amount of heat change during the formation of a solution (ii) Henry’s law can also be applied by expressing the
depends mainly on two factors: solubility of the gas in terms of mass per unit volume.
™ Lattice energy: It is the amount of heat required to (iii) Mass of the gas dissolved per unit volume of a solvent
separate one mole of the ionic substances into its at a given temperature is directly proportional to
component positive and negative ions. This is an the partial pressure of a gas in equilibrium with the
endothermic process. solution.

P
4 W JEE (XII) Module-1 CHEMISTRY
to life. To avoid bends, as well as, the toxic effects of high
m ∝ p, m=K×p concentrations of nitrogen in the blood, the tanks used
by scuba divers are filled with air diluted with helium
where,
(11.7% helium, 56.2% nitrogen and 32.1% oxygen).
m = mass of gas dissolved in unit volume of solvent. (iii) At high altitudes, the partial pressure of oxygen is less than
p = pressure of gas in equilibrium with solution. that at the ground level. This leads to low concentrations
Where K is the constant of proportionality that depends of oxygen in the blood and tissues of people living at high
on nature of gas, temperature and unit of pressure. altitudes or climbers. Low blood oxygen causes climbers
to become weak and unable to think clearly, symptoms of a
condition known as anoxia.
Characteristics of Henry’s Law Constant (KH)
Effect of Temperature & Le-Chatelier’s Principle
(i) Unit same as those of pressure i.e. torr or bar.
Solubility of gases in liquids decreases with rise in temperature.
(ii) Different gases have different values of KH. When dissolved, the gas molecules are present in liquid phase and
the process of dissolution can be considered similar to condensation
Partial pressure of HCl/torr

1000
and heat is evolved in this process. We have learnt that dissolution
process involves dynamic equilibrium and thus must follow Le
Chatelier’s principle. As dissolution is an exothermic process, the
500
slope = KH solubility should decrease with increase of temperature.

™ KH values for both N2 and O2 increase with increase


0.01 0.02 of temperature indicating that the solubility of gases
Mole fraction of HCl in its increases with decrease of temperature. It is due to this
solution in cyclohexane reason that aquatic species are more comfortable in
cold water rather than in warm water.
Figure: Plot of p Vs X for solution of HCl in cyclohexane.
(iii) The KH value of a gas is different in different solvents and
it increases with the increase in temperature.
(iv) Higher the value of KH of a gas, lower will be its solubility.
(v) Plot of p Vs X is a straight line passing through the origin
with slope equal to KH Example 4. If N2 gas is bubbled through water at 293K,
how many millimoles of N2 gas will be dissolved in 1L of
Limitation of Henry’s Law water.
Henry’s law is valid only for following conditions: Assume partial pressure of N2 = 0.987 bar.
(i) The pressure of gas is not too high. (Henry’s Law constant for N2 at 293K is 76.48 k bar)
(ii) The temperature is not too low.
Sol. PN 2 = K H .X N 2
(iii) The gas should not undergo any chemical reaction with the
solvent. 0.987 bar
X=
N
= 1.29 × 10−5
(iv) The gas should not undergo dissociation/association in
2 76480 bar
solution. 1L water contains 55.5 moles of water. Let, n be the
number of moles of N2 in solution
Application of Henry’s Law
n mol n
Several applications in biological and industrial phenomenon: =XN ≈
2 n mol + 55.5mol 55.5
(i) To increase the solubility of CO2 in soft drinks and soda
water, the bottle is sealed under high pressure. = 1.29 × 10–5
(ii) Scuba divers must cope with high concentrations ∴ n = (1.29 × 10–5 × 55.5) mol
of dissolved gases while breathing air at high = 7.16 × 10–4 mol
pressure underwater. Increased pressure increases the
= (7.16 × 10–4 × 1000) m mole
solubility of atmospheric gases in blood. When the
divers come towards surface, the pressure gradually = 0.716 m mole
decreases. This releases the dissolved gases and leads Example 5. Henry’s Law constant of CO2 in water at 298 K
to the formation of bubbles of nitrogen in the blood. is 5/3 k bar. If pressure of CO2 is 0.01 bar, then find its
This blocks capillaries and creates a medical condition solubility in terms of mole fraction.
known as bends, which are painful and dangerous

Solutions 5
Sol. P = KH.X 4. On cooling, solubility of a substance decreases.
Identify its solubility curve.
or, PCO2 (g) = K H .X CO2
(a) (b)
or, 0.01 = (5/3) × 1000 × X CO2

Solubility

Solubility
∴ X CO2 = 6 × 10–6
Example 6. Identify following statement as true or false
and identify correct match.
(i) Cooling favours the solubility of exothermic solutions.
Temperature Temperature
(ii) Heating favours the solubility of endothermic solutions.
(c) (d)
(iii)
(i) (ii)

Solubility

Solubility
(a) T F
(b) F T
(c) T T
(d) F F Temperature Temperature

Sol.
(i) We know that solubility of exothermic solutions
decreases on increase of temperature (heating).
VAPOUR PRESSURE
Therefore the given statement is true. When a liquid is placed in a closed container, it evaporates and
(ii) We know that solubility increases with increase of converts into vapour. After some time, the liquid and the vapour
temperature (heating). Therefore the given statement reaches equilibrium, i.e., the rate of evaporation becomes equal to
is true. the rate of condensation. At this point of time, the pressure exerted
by the vapours of the liquid on the surface of the liquid and on the
(iii) (c) T, T
walls of the container is called the vapour pressure of the liquid.

2. The boiling point of C6H6, CH3OH, C6H5NH2 and


C6H5NO2 are 80º, 67°, 180° and 210°C respectively. Which
will show highest vapor pressure at room temperature?
Does the vapour pressure of a liquid depend on the volume of the
(a) C6H6 (b) CH3OH
vessel, shape of the vessel or the amount of liquid taken in the
(c) C6H5 NH2 (d) C6H5 NO2 container?
3. Solubility of a substance remains unchanged on heating These questions can be answered by involving the concept of
or cooling. Identify its solubility curve. equilibrium constant. For the equilibrium, liquid  vapour,
(a) (b) the KP for the equilibrium would be equal to Pvap, since
this is the pressure exerted by the vapours at equilibrium
Solubility

Solubility

(by definition). We know Kp depends only on temperature. This


means that vapour pressure of a liquid will also be constant at a
given temperature and will depend only on temperature and the
Temperature Temperature nature of liquid.
(c) (d) Therefore, it should be noted that the vapor pressure of a liquid
is independent of whatever be the volume or shape of the vessel,
or the amount of liquid in it (assuming it to be more than the
Solubility

Solubility

minimum amount required).


Moreover, on addition of any substance to the liquid, as long as the
Temperature
substance does not dissolve in the liquid and completely covers
Temperature
the surface, the vapour pressure of a liquid remains constant.

P
6 W JEE (XII) Module-1 CHEMISTRY
Factors affecting vapour pressure
P = P°A+ P°B
(a) Temperature: Vapour pressure ∝ Temperature
where P is the total vapor pressure, and P°A, P°B are the vapor
pressures of the two pure liquids A and & B. If we let X′A and
X′B be the mol fractions of the two constituents in the vapor,

Vapour pressure
then P°A = X′AP, P°B = X′BP, and hence,
PA° X ′A P X ′A
= =
P° X ′B P X ′B
B

Temperature But X′A = nA/(nA + nB), and X′B = nB/(nA + nB), where nA and
Higher the temperature, higher will be the vapour pressure nB are the number of moles of A and B in any given volume
because a larger fraction of molecules have sufficient of vapor.
kinetic energy to escape. The total vapor pressure exerted by a mixture of immiscible
ADVANCED LEARNING liquids at a given temperature is the sum of the vapor pressures
of the individual components at that temperature.
Clausius-Clapeyron Equation
The boiling point of any system is the temperature at which the
P ∆H  1 1 
2.303 =
log 2  −  total vapor pressure is equal to the confining pressure. Since
P1 R  T1 T2 
the two liquids together can reach any given total pressure at a
Here P1 and P2 are V.P. at temperature T1 and T2 respectively lower temperature than either liquid alone, it must follow that
and ∆H = Enthalpy of evaporation / vapourisation and is always any mixture of two immiscible liquids must boil at a temperature
positive, thus if T1 < T2, then P1 < P2. lower than the boiling point of either of the two liquids.
(b) Nature of liquid: Ratio of Distillate to Residue Compositions

Vapour pressure of liquid ∝ The relative proportions of the two liquids in the distillate can
1 be calculated on the basis that the number of moles of each
The strength of intermolecular forces component present in the vapour phase is proportional to its
acting between molecules vapour pressure. If n′A and n′B are the number of moles of the
For example, CCl4 has higher vapour pressure because of components A and B in the vapour phase, then
the weak intermolecular forces acting between its molecules n′A/n′B = p°A/n°B (1)
than water which has stronger intermolecular forces acting
between water molecules. If wA and wB represent the actual masses of the two
ADVANCED LEARNING components in the distillate and MA and MB their respective
molecular masses, then Eq. (1) may be written as
DETERMINATION OF VAPOUR PRESSURE
OF A LIQUID w A n A M A PAº M A w pº M
= = º ⇒ A= Aº A
Experimental determination of vapour pressure involves two w B n B M B pB M B w B pB M B
methods:
Weight fraction of A in the distillate is
a. Static method:
wA= PAoMA/ ( PAoMA + PBoMB)
1. Barometric method
2. Manometric method
b. Dynamic method or Ostwald Walker method (to be RAOULT‘S LAW FOR MIXTURE
discussed later):
OF TWO VOLATILE LIQUIDS
VAPOR PRESSURES OF IMMISCIBLE
LIQUIDS
Since immiscible liquids are mutually insoluble, addition of one
liquid to the other does not affect the properties of either liquid.
Hence each will behave as if the other were not present.
Consequently, in a mixture of two immiscible liquids, each
will exert the vapor pressure corresponding to the pure liquid
at the given temperature, and the total vapor pressure above the mole fractions of A and
mixture will be the sum of the vapor pressures of the two pure B in liquid solution
constituents, namely,
Figure: Let A, B be two volatile liquids

Solutions 7
Partial pressure of A = PA and Partial pressure of B = PB
RAOULT’S LAW IS A SPECIAL
According to Raoult’s law (experimentally)
CASE OF HENRY’S LAW
PA ∝ XA
According to Henry’s law
PA = XAPAº
p = kHx ...(i)
Here PAº & PBº are vapour pressure of pure liquid A & B at
where, x is the mole fraction of the gas (solute) in the solution
given temperature which is in contact with the gas above having pressure equal to p.
Similarly. PB ∝ XB Here, it may be mentioned that Raoult’s law holds good for ideal
∴ PB = XBPBº solutions in which both solute and solvent are volatile liquid. In
the above example, we can regard p as the vapour pressure of
if PAº > PBº volatile solute (gas) when it is present to the extent of x2 mole

∴ A is more volatile than B fraction in the solution.
Applying Raoult’s law

∴ boiling point of A < boiling point of B
P = p°x ...(ii)

∴ According to Dalton’s law, where p° is the vapour pressure of pure solute (dissolved gas in the
PT = PA + PB = XAPAº + XBPBº present case) at the temperature of the solution.
Eqs. (i) and (ii) are quite similar. In fact, both become identical if
Graphical Representation kH becomes equal to p°. Therefore. Raoult’s law can be taken as a
special case of Henry’s law.
PA= XAPAº and PB = XB PBº
PT = XAPAº + XB PBº IDEAL SOLUTIONS
PT = XAPAº + (1 – XA) PBº = ( PAº – PBº ) XA + PBº The mixtures which follow Raoult’s law at all temperatures
and at all compositions are known as ideal mixtures / ideal
PT = (1 – XB) PAº + XBPBº = ( PBº – PAº ) XB + PAº
solution.
PT = PA + PB Characteristic of an ideal solution:
™ Ideal solution are obtained only when the forces of attraction
between the liquid molecules are exactly of same nature and
Here almost of same magnitude.
PAº > PBº i.e. A --- A ≈ A --- B ≈ B --- B
(A is more volatile than B) ™ ∆Hmix = 0, ∆Vmix = 0
™ ∆Smix = + ve
™ ∆Gmix = – ve

Example:
Relation between Mole Fraction of (1) Benzene + Toluene.
Vapour Phase and PT (2) n-Hexane + n-Heptane.
XA′ = mole fraction of A in vapour above the liquid / solution. (3) C2H5Br + C2H5Ι.

XB′ = mole fraction of B in vapour above the liquid / solution. (4) Chlorobenzene + Bromobenzene;
Dalton’s law for gaseous mixture,
PA = X'A PT ™ In case of ideal solution the vapour phase is richer with
more volatile component (i.e., the one having relatively
Raoult’s law when liquid and vapour are in equilibrium greater vapour pressure) in comparison to that in
PA = XAPAº = X'A PT solution phase.

PB = XBPBº = X'B PT ADVANCED LEARNING

X 'A PT X 'B PT CONDENSATION OF VAPOURS OF


X A + XB = 1 = +
PAο PBο SOLUTION
When the vapours of ideal solution (containing liquids
1 X X′
= °A + °B A and B) is condensed , the composition of liquids A and B in the
PT PA PB condensate remains same. The vapours over condensate can

P
8 W JEE (XII) Module-1 CHEMISTRY
again be recondensed and the composition of A and B in
condensate (2) remains same as it was in vapour phase over
condensate (1).
Example 7: The vapour pressures of ethanol and methanol
are 44.5 mm Hg and 88.7 mm Hg respectively. An ideal
Vapour
solution is formed at the same temperature by mixing 60 g
of ethanol with 40 g of methanol. Calculate the total vapour
Liquid
pressure (in mm Hg) of the solution and mole fraction of
methanol in the vapour.
Solution Condensate(1) Condensate(2) 60
Sol. Number of moles of C2H5OH = = 1.304
46
FRACTIONAL DISTILLATION 40
=
Number of moles of CH3OH = 1.25
Repeated boiling of any mixture of two volatile liquids, as 32
in fractional distillation, always results in the separation of
1.304
the mixture. The liquid in the mixture with the lower boiling Mole fraction of C2H5OH = = 0.5107
1.304 + 1.25
point (and higher vapour pressure) comes off the top of the
fractionating column, while the other liquid is left in the flask. 1.25
Mole fraction of CH3OH = = 0.4893
Mixtures which behave like this with both components obeying 1.304 + 1.25
(or nearly obeying) Raoult's law are called ideal mixtures and Partial pressure of C2H5OH = XA.PºA
can be completely separated by fractional distillation.
= (0.5107 × 44.5) mm Hg
Examples are: liquid air (a mixture of nitrogen and oxygen)
and petroleum. = 22.73 mm Hg

For Ideal Solution Partial pressure of CH3OH = X B .PBº


= (0.4893 × 88.7) mm Hg
TA° Vapour = 43.40 mm Hg
Total vapour pressure of solution = (22.73 + 43.40) mm Hg
Liquid = 66.13 mm Hg
TB°
Mole fraction of CH3 OH in vapour
pressure
Vapour

a′′ Partial pressure of CH3OH


=
(b) (a) (a′) Total vapour pressure
cB = 1
cA = 1 Mole Distillate
fraction 43.40
Residue from richer in B = 0.6563
=
66.13
distillation of a Mixture composition
‘a’ richer in A
Graph for boiling point vs composition for ideal solution Example 8: Liquids ‘A’ and ‘B’ form ideal solution. At
™ Point 1: Mixture composition (a) is taken richer in a 80°C, PAº = 0.4 bar and PBº = 0.8 bar. All the vapour above
component (A) the liquid solution containing equal moles of both the
™ Point 2: Mixture composition (a) boils at Tb(a) temperature liquids at equilibrium is collected in another empty vessel
™ Point 3: Distillate (a′) of (a) is the mixture composition and condensed. Now, the condensate is heated to 80°C
richer in component B. and all the vapours above the liquid solution at equilibrium
™ Point 4: ‘b’ is residue during distillation of mixture is again collected in another empty vessel and condensed.
composition ‘a’. What is the mole fraction of ‘B’ in new condensate?
Hence such solution can be successfully solution separated Sol. For the first condensate,
into pure A and pure B. nB X B YB X B . PBº nB PBº
= ; = = ×
Successive distillation of a′ give a′′ and then finally pure B, nA X A YA X A .PAº nA PAº
as distillate.

Solutions 9
NON-IDEAL SOLUTIONS
For second condensate,
2 In terms of Raoult’s law, a non-ideal solution is defined as the
nB′′ X ′′B YB′  YB  PBº nB  PBº 
= ; =   = ×  one which does not obey Raoult’s law. These are divided into two
nA′′ X ′′A YA′  YB  PAº nA  PAº 
2 types as explained below:
x  0.8 
= = .  4 [As moles of both liquids are equal]
x  0.4  (1) N
 ON-IDEAL SOLUTIONS SHOWING POSITIVE
nB′′ nB′′ 4n ′′A
⇒ XB =
=4 ⇒ n ′′B =4n ′′A = DEVIATIONS
nA′′ nA′′ + nB′′ nA′′ + 4nA′′
These can be defined on the basis of
4
∴ Mole fraction of B= = 0.8 (a) Molecular interactions
5
Example 9: Vapour pressure of CH3 Cl and CH2 Cl2 are A non-ideal solution showing positive deviation is defined as the
540 mm Hg and 402 mm Hg respectively. 101 g of CH3 Cl one in which the intermolecular forces of interactions between
and 85 g of CH2Cl2 are mixed together. Determine the molecules is weaker than the interactions found in either of
(i) The pressure (in mm Hg) at which the solution the pure components.
starts boiling. (b) Thermodynamics
(ii) Molar ratio of solute vs solvent in vapour phase in
equilibrium with solution. A non-ideal solution is defined as the one in which there is a slight
Sol. (i) Solution boils when atmospheric pressure becomes increase in volume on mixing and absorption of heat takes place
equal to vapour pressure. on mixing. ∆Vmixing = positive, ∆Hmixing = positive.
∴ Boiling atmospheric pressure = vapour pressure In general, the trend of molecular interactions in the molecules are
° °
= PA X A + PB X B hydrogen bonding > dipole−dipole interactions > van der Waal’s
 2  1 interaction (for compounds having similar molar mass). Hence,
Total pressure =  540 ×  +  402 × 
 3  3 replacement of any one of the stronger interactions by the weaker
= (360 + 134) mm Hg one would result in positive deviation.
= 494 mm Hg ™ PTexp. > ( XAPº A + XBPºB)
  (ii) Here, the solute is CH2Cl2
A − − − −A
402 × 13 134
X= ′CH Cl = ™ or > A ---- B
2 2
494 494 B − − − −B
→

2
540 × 3 360
X= ′CH Cl = Weaker force of attraction
3
494 494
n′CH 2Cl2 nsolute 134 ™ ∆Hmix = +ve
now, = = = 0.372
n′CHCl3 nsolvent 360 ™ ∆Vmix = +ve (1L + 1L, then final volume of solution is greater

Example 10: At 25°C, the vapour pressure of methyl then 2L)


alcohol is 96.0 torr. What is the mole fraction of CH3OH in ™ ∆Smix = +ve
a solution in which the (partial) vapor pressure of CH3OH ™ ∆Gmix = –ve
is 23.0 torr at 25°C?
PCH3OH 23 Example:
Sol. XCH OH = 0 = = 0.24 (i)
Acetone (dipole) + carbon disulphide (van der Waals)
3 PCH3OH 96
(ii)
Acetone (dipole) + ethyl alcohol (hydrogen-bond)
(iii)
Acetone (dipole) + benzene (van der Waals)
(iv)Carbon tetrachloride (van der Waals) + Chloroform
(dipole)
(v) Methyl alcohol + water
5. Two liquids A and B form an ideal solution. At 300K, the (vi) Ethyl alcohol + water
vapour pressure of a solution containing 1 mole of A and In the given examples from (i) to (vi), the molecules possessing
3 mole of B is 550 mm Hg. At the same temp, if one more weaker interactions get in between the molecules possessing
stronger one and thereby reducing the overall interactions.
mole of B is added to the solution, the vapour pressure
In examples (v) and (vi), though both the components have
of the solution is increased by 10 mm Hg. Determine the hydrogen bonding but the extent of hydrogen bonding in
vapour pressure of A and B in the pure state (in mm Hg). the case of water is more than that in the case of alcohols
(methanol or ethanol) or alcohol-water.

P
10 W JEE (XII) Module-1 CHEMISTRY
Graphical Representation ™ ∆Vmix = –ve (1L+1L, then final volume of solution is less
V.P. = max. then 2L)
B.P. = min. ™ ∆Smix = +ve
P>
PA + ™ ∆Gmix = –ve
Vapour pressure P B
Examples
P (i) Chloroform (dipole) + acetone (dipole)
>
P AX
0 A A
P (ii) Acetone(dipole) + aniline (weak hydrogen bond)
> B 0X
PA (iii) Pyridine (van der Waal’s) + Acetic acid (hydrogen
B
bonding)
XA = 0 Mole fraction XA = 0 (iv) HCl + water
XB = 1 XB = 1 (v) HNO3 + water
Cl
CH3
(vi) acetone + chloroform; Cl C H O C
Cl CH3
In the above examples from (i) to (ii), the weak interactions are
being replaced by strong hydrogen bonds, thus reducing the
escaping tendency of the molecules. In case of (iii), one of the
constituents is basic while the other is acidic. Whereas (iv) and
(v) involve an aqueous solution of a strong volatile acid such as
a halogen acid. In these cases, non-volatile ions are formed as a
result of interaction of ions of the acid with water.
Positive deviation from Raoult’s law
Graphical Representation
For one intermediate composition, the total vapour pressure of
such a solution will be the highest and the boiling point will be V.P. = min.
the lowest. This solution acquires the property of boiling at a
B.P. = max.
constant temperature and its composition in liquid phase remains
Vapour pressure

unchanged because the composition in vapour phase is also same + PB


P < PA
as in liquid phase. Liquid mixtures, which boil out without any 0XA
change in the composition are called azeotropes or azeotropic P > PA
A > PA
mixtures. In case of positive deviation from Raoult's law, we get P
B 0X
minimum boiling azeotrope. A

(2) NON-IDEAL SOLUTIONS SHOWING NEGATIVE XA = 0


XA = 0 Mole fraction
DEVIATIONS XB = 1 XB = 1
These can be defined on the basis of
(a) Molecular Interactions
Solutions in which forces of interactions between the components Maximum boiling
azeotropic mix
is stronger than that in between both the pure components are the P
ones showing negative deviation from Raoult’s law.
(Azeotropic composition)
(b) Thermodynamics
X
In the case of such solutions, a slight decrease in volume and
evolution of heat takes place on mixing i.e.,
∆Vmixing = negative and ∆Hmixing = negative Negative deviations from Raoult’s law
™ PT exp < XAPºA + XBPºB For one intermediate composition, the total vapour pressure of
A − − − −A such a solution will be the lowest and the boiling point will be the
™ and <A ------ B highest. This solution acquires the property of boiling at a constant
B − − − −B
→

temperature and its composition remains unchanged because the



composition in vapour phase is also same as in liquid phase.
stronger force of attraction.
In case of negative deviation from Raoult's law, we get maximum
™ ∆Hmix = –ve
boiling azeotrope.

Solutions 11
ADVANCED LEARNING Point 2: When mixture composition c is taken and boiled.
TEMPERATURE VERSUS COMPOSITION (i) Distillate is c′ which is richer in component B and
CURVES successive distillation of distillates lead to solution
becoming pure ‘B’
For non Ideal Solutions
(ii) The residue of ‘c’ is ‘e’ successive distillation of ‘e’
A) +Deviation solution or (–ve boiling azeotropes) makes the residue richer in ‘D’ which is +ve boiling
When temperature versus composition graphs are drawn azeotropic mixtures.
for +ve deviation solution we get

TA° Vapour
Liquid V
TB° Example 11: The boiling point of an azeotropic mixture of
Vapour L water and ethanol is less than that of water and ethanol. The
pressure D mixture shows
e (a) No deviation from Raoult’s Law
b a a′ c′ (b) Positive deviation from Raoult’s Law
c cB = 1
cA = 1 d (c) Negative deviation from Raoult’s Law
Mole
fraction (d) That the solution is unsaturated
(Graph for boiling point vs to composition for non-ideal Sol. Positive deviation is noticed, because, water-ethyl
solution showing +ve deviation) alcohol interaction is weak, hence boiling point is low.
Point 1: Here when mixture composition ‘a’ is boiled, then That means higher vapour pressure. Hence, positive
the distillate (a′) becomes more rich in ‘B’ but, successive deviation is shown.
distillation of a′ will finally lead to compositions (‘d’) where
vapour phase and liquid phase compositions are same. Hence
d(D) is a solution which cannot be separated into pure A and B.
Hence it is a –ve boiling azeotropic mixture. But the residue ‘b’
becomes richer in ‘A’.
Point 2: When mixture composition ‘c’ is boiled then distillate
is (c′) richer in ‘A’ but successive distillation makes it to 6. A non-ideal solution was prepared by mixing 30 mL
composition d(D) i.e. –ve boiling azeotropic mixture. chloroform and 50 mL acetone. The volume of mixture
will be ________ mL.
The distillate is ‘e’ richer in (B) component
B) –ve Deviation solutions (+ve Boiling azeotropic)
When temperature versus composition are drawn for the
solutions showing –ve deviation the graph obtained is COLLIGATIVE PROPERTIES AND
CONSTITUTIONAL PROPERTIES
D V
V
TB° Colligative Properties
TA° The properties of the solution which are dependent on the total
L
d no. of solute particles or total concentration of solute particles in
a′ a b c c′ the solution and are not dependent on the nature of solute particle
XA = 1 d e XB = 1 i.e., shape, size, charge etc. of the particles.
Mole fraction
There are 4 colligative properties of solution.
(Graph of Boiling point Vs composition for non-ideal solution
–ve deviation)  ∆P 
™ Relative lowering in vapour pressure  
Point 1: When mixture composition ‘a’ is taken, then on boiling.  Pº 
(i) Distillate a′ is obtained which is richer in component ™ Elevation in boiling point (∆Tb)
A
™ Depression in freezing point (∆Tf)
(ii) Residue is ‘b’
Successive distillation of ‘b’ leads to azeotropic ™ Osmotic pressure (π)
solution D. (+ve boiling azeotropic mixture)
(iii) Successive distillation of a′ will lead to pure (A) To be more accurate, the colligative property depends upon the
fraction of solute and solvent particles in solution.

P
12 W JEE (XII) Module-1 CHEMISTRY
(Here m = Molar mass of solute and M = Molar mass of solvent)
RELATIVE LOWERING IN VAPOUR
P° − PS w M 1000 w 1000 M
PRESSURE (RLVP) A
PS
=
m
× ×
W 1000
=
m
×
W
×
1000

Lowering in Vapour Pressure of a Solution P ° – Ps M


A = (molality)× (M = molar mass of solvent)
Vapour pressure of a solution of a non-volatile solute Ps 1000
(solid solute) is always found to be less than the vapour pressure This indicates that equimolecular quantity of any non-volatile
of pure solvent.
solute dissolve in same quantity of the same solvent will produce
Reason the equal lowering of vapour pressure. Hence, this is colligative
Some of the solute molecules will occupy some surface area of property.
the solutions, so tendency of the solvent particles to go into the
vapour phase is slightly decreased because A is volatile solvent
Key Note
and B is non-volatile solute. Vapour pressure of B = 0 (i.e. PºB = 0)
because B is non-volatile.
™ (PºA– PA) is the lowering of vapour pressure whereas
 PA0 − PA 
 0  is called relative lowering of vapour
 PA 
pressure.
™ Mole fraction of solute
Vapour pressure of pure solvent − Vapour pressure of solution
=
Vapour pressure of pure solvent

™ Vapour pressure of solutions having non volatile solute


will always be less than that of solvent, however vapour
pressure of solutions with volatile solute may be more or
Figure: PS < PoA less than that of pure solvent.
From Raoult’s law
PS = PºA XA + PºB XB ADVANCED LEARNING
PS = PºA XA  (As PºB = 0, So PºB XB= 0)
Dynamic Method
PS = PºA (1 – XB)
Measurement of Relative Lowering of Vapour Pressure
PS = PºA – PºA XB using Ostwald and Walker’s Method
PºAXB = PºA – PS (Lowering in V.P. = PºA – PS= ∆P) Ostwald and Walker apparatus is the arrangement to calculate
PºA – PS ∆P ∆P relative lowering in vapour pressure. In this method, a stream
XB = = (Relative lowering in V.P. = ) of dry air is bubbled successively through (i) the solution
PºA PºA PºA
(ii) the solvent and (iii) a reagent which can absorb the vapours
™ Raoult’s law: (For non – volatile solutes)
of the solvent. Generally solvent is water and thus the reagent
Experimentally relative lowering in Vapour pressure = mole used is anhydrous CaCl 2.
fraction of the non volatile solute in solution.
PºA – Ps
RLVP =
PºA
n
RLVP = XSolute =
n+N
PºA n+N N CaCl 2-U-tube
⇒ = = 1+
PºA – Ps n n
(Here n = Moles of solute and N = Moles of solvent) The first three bulbs contain a weighed amount of the solution
and the next three bulbs contain a weighed amount of the pure
PºA Pº – PºA + Ps Ps
N = –1 = A = solvent. A weighed amount of anhydrous CaCl2 is taken in the
n PºA – Ps PºA – Ps PºA – Ps U tube. All the bulbs must be kept at the same temperature and
Pº – Ps PºA – Ps w dry air must be bubbled gradually to ensure that it gets saturated
n M
A = ; = × with the vapours in each bulb.
Ps N Ps m W

Solutions 13
The dry air, as it passes through the solution, takes up an amount
of vapour which is proportional to the vapour pressure of the Example 14: Dry air is passed through a solution containing
20 g. of an organic non-volatile solute in 250 ml of water.
solution at the prevailing temperature. This moist air when
Then the air was passed through pure water and then
passes through water (solvent), it takes up a further amount through a U-tube containing anhydrous CaCl2. The mass
of vapour which is proportional to the difference in vapour lost in solution is 26 g and the mass gained in the U-tube is
26.48 g. Calculate the molar mass (in g/mol) of the organic
pressure of pure solvent and vapour pressure of the solution.
solute. (dsolution = 1 g/ml)
(i) Loss in mass of solution ∝ vapour pressure of solution
Sol. Loss in mass in solution = 26g
∝ PS Gain in mass in U-tube = 26. 48 g
(ii) Loss in mass of solvent ∝ vapour pressure of pure Therefore, Loss in mass in solvent = 0.48 g
solvent - vapour pressure of solution ∝ Pº – PS Loss in mass of solvent p° − p
We know that = =
(iii) Loss in mass of solution + loss in mass of solvent gain in ma ss of U − tube P°
∝ PS + Pº – PS ∝ Pº 0.48 P° − P
= = mole fraction of solute in the
This moist air is then passed through CaCl2 taken in U-tube. 26.48 P°
solution
The CaCl2 tubes are weighed at the end of the experiment. The 0.48 n solute n solute
= =
gain in mass should be equal to the total loss in mass of the 26.48 n solute + n solvent n solute + 250 / 18
solution and solvent bulbs which is proportional to Pº.
250 × 0.48
ο nsolute =
18 × 26
i.e., loss in mass of solvent = P – PS
gain in mass of CaCl 2 Pο 25 × 18 × 26
Msolute = = 78 g / mol
Important. When a dry gas is bubbled slowly through a pure 250 × 0.48
liquid or a solution, the gas gets saturated with respect to that Example 15: Dry Air was passed through a solution
liquid or solvent. e.g. If V litres are bubbled through a pure containing 20 gm. of substance in 100 gm of water and
then through pure water. The loss in mass of the solution
liquid having molecular weight M and a vapor pressure P° at was 2.9 gm and that of pure water was 0.06 gm. Calculate
that temperature, T, and if this leads to a reduction in w grams the molar mass (in g/mol) of the substance.
of the liquid then P° = wRT/MV Sol. Ps ∝ loss in mass of solution
p0 – Ps ∝ loss in mass of pure water
Then, Ps ∝2.9
p0 – Ps ∝ 0.06
or, P0 ∝ (2.9 + 0.06) ∝ 2.96 = 3
Now,
Example 12: 1 mole of a non-volatile solute is dissolved p0 − ps 0.06
in 2 moles of water. The vapour pressure of the solution = = x2
p0 3
relative to that of water is ________
n2 n 2 w 2 / M 2 w 2 / M1 20 × 18
≈= = = 100 × M
Pº − Psolution 1 1 n1 + n 2 n1 w1 / M 2 w1 / M 2
Sol. solvent = = 2
º
Psolvent 1+ 2 3 or, M2 = 180 gm/mole
P 1
solution =1 − =2 Example 16: The vapour pressure of water at 293 K is
º
Psolvent 3 3
17.54 mm of Hg and the vapour pressure of solution at
Example 13: The vapour pressure of pure liquid solvent A the same temperature is 17.35 mm of Hg. The solution is
is 0.80 atm. When a nonvolatile substance B is added to the prepared by dissolving 100 gm. of non-volatile solute in one
solvent its vapour pressure drops to 0.60 atm. What is the litre water. Calculate the molar mass (in g/mol) of solute.
mole fraction of component B in the solution? ∆p
Sol. = X2
Sol. p10

PA0 − Psol Now, P10 = 17.54 mm of Hg


0.80 − 0.60
XB = = = 0.25 Dp = (17.54 – 17.35) = 0.19 mm of Hg
PA0 0.80

P
14 W JEE (XII) Module-1 CHEMISTRY
equal to Pext, we have to increase the temperature of solution by
n2 n w 2 / M2 some amount in comparison to that of pure solvent.
Now, X 2 ≈ ≈ 2 =
n1 + n 2 n1 n1 1atm

Vapor Pressure
wt. of solute in gm 1000 Psolv
= = 55.56 T

(atm)
n1 = Mole wt. of solvent 18
b
Psoln
w2 = mass of solute in gm = 100 gm bpsolv bpsoln
M2 = Molar mass of solute Temperature (°C)
0.19 100 1
= × Graphical Representation
17.44 M 2 55.56
⇒ M2 = 166 g/mol

™ If solution are dilute then BE and CD can be approximated


7. The vapour pressure of pure benzene at a certain temp as straight lines.
is 640 mm Hg. A non-volatile solid weighing 2.175 g
is added to 39.0 g of benzene. The vapour pressure of ∴ ∆ABE and ∆ACD will be similar. Using property of similar
solution is 600 mm Hg. What is molecular weight triangles, we can write
(in g/mol) of solid substance? AB AE ∆Tb1 ∆P1
= ⇒ =
8. Vapour pressure of solution containing 6 gm of AC AD ∆Tb2 ∆P2
non-volatile solute in 180 gm of water is 20 Torr.
∴ ∆Tb α ∆P
If 1 mole of water is further added, then the vapour
pressure increases by 0.02 Torr. Calculate vapour n
∆Tb ∝ ∆P ∝
pressure (in Torr) of pure water and molecular weight N
of non-volatile solute? n w×M
∴ ∆Tb ∝ = (for a given solvent P° and M = constant)
9. In Ostwald and Walker’s Apparatus, if the loss in N m× W
wt. of solution is 6 unit and loss in wt of solvent is w K×w
∆Tb ∝ or ∆Tb =
3 unit, then calculate the mole fraction of the solute. m× W m× W
Tbs → b ⋅ p ⋅ of solution
∆ Tb = TbS – Tb°  ο
Tb → b ⋅ p ⋅ of solvent
ELEVATION IN BOILING POINT (∆Tb) w
If = 1 mole and W = 1000 gm, then ∆Tb = Kb
m
Boiling Point of a Liquid
The temperature at which vapour pressure of a liquid becomes K
∴ = Kb (Molal
 elevation constant or Ebullioscopic
equal to the external pressure present at the surface of the liquid is 1000 constant)
called boiling point of liquid at that pressure.
w 1000
Normal Boiling Point ∆Tb = × × Kb
m W
The temperature at which boiling occurs when the external pressure
is exactly 1 atm is called the normal boiling point of the liquid. (Tb) ∴ ∆Tb = molality × Kb ∴ ∆Tb ∝ molality
Hence, elevation in boiling point (∆Tb) is a colligative
H2O()  H2O(g) ∆H = +ve property.
Standard Boiling Point Using thermodynamics

The temperature at which boiling occurs when the external RTb 2


™ Kb =
pressure is exactly 1 bar is called the standard boiling point of 1000× L vap
the liquid. (Tb) ∴ LVap = latent heat of vapourisation in cal/g or J/g
Elevation in Boiling Point R = 2 cal mol–1 K–1 or 8.314 J mol–1 K–1.
Since vapour pressure of solution is smaller than the vapour Tb = boiling point of liquid (in kelvin)
pressure of pure solvent at any temperature, hence to make it Unit of Kb = K kg mol–1

Solutions 15
RTb2 M Key Note
Kb =
1000 × ∆H vap

∆Hvap – molar enthalpy of vapourisation in cal/mole or ∆Tb(1) K b(1)
(i) Also = when concentration is same but
J/mole ∆Tb(2) K b(2)
Here, M is molecular wt. of solvent solvent different
 ∆H vap  ∆Tb(1) m(1)
Lvap =   (ii) ∆T =
 M  b(2) m when solvent is same but concentration
2

different
DEPRESSION OF FREEZING ∆Tb(1) M o(2)
POINT (∆Tf) (ii) ∆T =
b(2) M o(1) ⇒ (Mo = Molar mass of solute) when
solvent is same and amount of solute is same
Freezing Point
Temperature at which vapour pressure of solid phase becomes
equal to vapour pressure of corresponding liquid phase is called ANTIFREEZE SOLUTIONS
freezing point of liquid. Water is used in radiators of vehicles as cooling liquid. If the
vehicle is to be used at high altitudes where temperature is sub-
Normal Freezing Point zero. water would freeze in radiators. To avoid this problem. a
The temperature at which the liquid/solution is in equilibrium solution of ethylene glycol in water is used in radiators which will
with its solid state at an external pressure of 1 atm is called normal lower the freezing point lower than zero.
freezing point of the liquid. (Tf)
Kb, Kf for various Solvents (k.kg mol– 1)
H2O()  H2O(s) ∆H = –ve
Solvent B.P. Kb F.P. Kf
Standard Freezing Point Acetic acid 118.1°C 2.93 17°C 3.9
The temperature at which the liquid/solution is equilibrium with Benzene 80.2°C 2.53 5.4°C 5.13
its solid state at an external pressure of 1 bar is called standard Water 100°C 0.53 0ºC 1.86
freezing point of the liquid. (Tf)
Graphical Representation
Pressure
Vapor

Example 17: The boiling point of pure acetone is 56.38ºC.


When 0.707 g of a compound is dissolved in 10 g of acetone,
there is b.p. elevation to 56.88°C, what is the M.W. (in g/mol)
™ For dilute solutions, BE and CD can be assumed to be straight of the compound? (K b(CH CO CH ) ) = 1.72 K kg mole–1
3 3

lines. Sol. ∆ Tb = Kb × m
∴ using similar triangles ∆Tf a ∆P 0.707 1000
= (56.88 – 56.38) =1.72 × ×
∴ ∆Tf = Kf . molality MB 10
™ Kf = molal depression constant = cryoscopic constant ∴ MB = 243.2 g/mol
RTf 2 RTf 2 M Example 18: If freezing point of a solution prepared
Kf = =
1000 × Lfusion 1000 × ∆H fusion from 1.25 g of non electrolyte and 20g water is
271.9 K, then molar mass of the solute (in g/mol) will be
For water, Tf = 273 K and Lfusion = 80 cal/gm
(Kf (H2O) = 1.86 K kg mol–1) (Tf° = 273.15 k)
2 × 273 × 273
Kf = = 1.86 K kg mol–1 Sol.   Tf = 271.9 K
1000 × 80
™ Now, at freezing point or below temperature only solvent   w = 1.25 g, W = 20 g, Kf = 1.86
molecules will freeze, no solute molecules will be present in ∆ Tf = Tºf – Tf = 273.15 – 271.9 = 1.25 K
the solid formed (solid phase will only contain pure solvent).
™ From room temperature to the state of freezing point of
w

∆T=
f × 1000 × K f
solution, we will always assume that the molality of solution m× W
will remain same but if the temperature of solution goes w × 1000 × K f
below its freezing temperature, then some solvent will freeze
∴ m=
suddenly to change the molality of solution, and the molality ∆T f × W
of remaining solution increases.

P
16 W JEE (XII) Module-1 CHEMISTRY
1.25 × 1000 × 1.86

m= = 93 g/mol.
1.25 × 20
Example 19: 18 g glucose (C6H12O6) is dissolved in
1 kg of water in a saucepan. At what temperature (in kelvin)
will the water boil (1.013 bar pressure)? Kb(H2O) (Tb° =
373.15)= 0.52 K kg mol –1.
w B × 1000
Sol. ∆ Tb = Kb ×
mB × w A
18 × 1000
∆ Tb = 0.52 ×
180 × 1000
Tb–T0 = 0.052 OSMOTIC PRESSURE (p)– When a solution is separated
from the pure solvent with the help of a semipermeable
Tb – 373.15 = 0.052 membrane. There occurs the flow of solvent molecules from the
Tb = 373.15 + 0.052 = 373.202 K pure solvent to the solution side. The flow of solvent molecules
from a region of higher concentration of solvent to the region
Example 20: A 0.5 percent aqueous solution of Potassium of lower concentration of solvent is termed as the phenomenon
chloride was found to freeze at 0.24°C. Calculate the Van’t of osmosis. This also happens when two solution of different
Hoff factor and degree of dissociation of the solute at this concentra­tions are separated by a semipermeable membrane.
As a result of osmosis a pressure is developed which is termed
concentration (kf for water is 1.86).
as osmotic pressure. It is defined as.
i × 1000k f w (1) The excess hydrostatic pressure which builds up as a result
Sol. ∆Tf = of osmosis is called osmotic pressure.
W.M
(2) The external pressure which must be applied on the solution
i × 1000 × 0.5 × 1.86 side in order to just stop the flow of the solvent molecule
0.24 = into the solution side through semipermeable membrane.
100 × 74.5
(3) Osmotic pressure of a solution is equal to the negative
0.24 × 100 × 74.5 pressure which must be applied to the sol­vent in order to
i= = 1.923; just stop the osmosis.
1000 × 0.5 × 1.86
⇒ osmotic pressure = hydrostatic pressure
i = 1 + ( n –1 ) α
p = hdg
1.923 = 1 + ( 2 –1 ) α h = increase in level in the tube of unit cross section.
1.923 = 1 + α d = density of solution,
α= 1.923 – 1 = 0.923 g = acceleration due to gravity
or α = 0.923 or 92.3 % OR

10. Pure benzene freezes at 5.45°C. Solution containing


7.24 g C 2H 2Cl 4 in 115.3 g benzene was observed
to freeze at 3.55°C. What is molal freezing point
(in °C/m) of benzene.

OSMOSIS AND OSMOTIC PRESSURE Van’t Hoff law for a dilute solution:
The osmotic pressure (p) of a solution is directly proportional to
OSMOSIS: Osmosis is defined as the spontaneous flow of
its concentration(C) when the temperature is kept constant.
solvent molecules through semipermeable membrane from pure
solvent side to the solution side or from a dilute solution to the Gay Lussac – Van’t Hoff law – p V = nST
concentrated solution. Here, p = Osmotic pressure
V = Volume of solution (in lit.)

Solutions 17
n = No. of moles of solute (i) When placed in water or hypotonic solutions, cell swell and
S= Molar solution constant (0.082 lit atm K–1 mol–1) burst (haemolysis).
T = Temperature (in K) (ii) When placed in hypertonic solutions, the fluid from the
n plant cells comes out and thus the cells contract in size
π = ST or p = CST
V (plasmolysis). e.g. When excess of fertilizers (like urea) are
n applied, plasmolysis takes place and plants dry up (wilt).
[ C = = molar concentration of solution]
V Hypotonic and Hypertonic Solutions
Important: Osmotic pressures can be determined quite accurately, A solution having lower osmotic pressure then other solution is
hence it is used in the determination of molecular weights of large said to be hypotonic with respect to other solution.
proteins and similar substances.
A solution having higher osmotic pressure than other solution
is said to be hypertonic with respect to other solution.
SEMIPERMEABLE MEMBRANE
™ A membrane which allows the passage of solvent molecules REVERSE OSMOSIS (R.O.)
but not that of solute.
e.g. Copper ferrocyanide Cu2[Fe(CN)6]; If external pressure greater than osmotic pressure is applied on the
The most frequently used semipermeable membrane in solution side, the flow of solvent molecules can be made to proceed
laboratory is that of copper ferrocyanide, Cu2[Fe(CN)6] from solution side towards pure solvent side, i.e., in reverse direction
because it is very strong and can withstand very high of the ordinary osmosis.
pressure. Remember that semipermeable membrane of Reverse osmosis is used for the desalination of sea water for
Cu2[Fe(CN)6] does not work in non-aqueous solution getting fresh drinking water.
because it gets dissolved in non-aqueous solvents. Other
synthetic semipermeable membrane is that of calcium
phosphate. Natural semipermeable membranes are
parchment paper, cell walls, pig’s bladder, skin surrounding
white of an egg, membrane surrounding RBCs etc.
Example 21: A cane sugar solution has an osmotic pressure
Key Note of 2.46 atm at 300 K. What is the molar concentration of
the solution?
(a) Exosmosis is the outward flow of water from a n
cell containing an aqueous solution through a Sol. π V = nST or π = ST = CST
V
semipermeable membrane, e.g. grape in NaCl solution.
π 2.46
(b) Endosmosis is the inward flow of water into a or=C = = 0.1 M
ST 300 × 0.0821
cell containing an aqueous solution through a
semipermeable membrane, e.g. grape in water.
™ If a number of solutes are present in the solutions and
p1, p2, p3 etc. are their individual osmotic pressures, then
Total osmotic pressure = p1 + p2 + p3 + .....
11. What is the osmotic pressure (in atm) of 12% w/v
solution of cane sugar (molecular weight 342) at 17°C.
12. A solution containing 8.6 g urea in one litre solution was
ISOTONIC OR ISO-OSMOTIC found to be isotonic with 0.5% (weight/volume) solution
SOLUTION of an organic, non-volatile solute at same temperature. The
molecular weight (in g/mol) of the solute is ______
Solutions which have the same osmotic pressure are termed
isotonic or iso-osmotic solutions. ABNORMAL COLLIGATIVE
π1 =π2 i.e., C1ST = C2ST; C1 = C2 PROPERTIES
If two such solutions are separated by a semipermeable membrane,
there will be net zero transference of solvent from one solution to It has been observed that difference in the observed and calculated
the other. molecular masses of solute is due to association or dissociation of
0.85% NaCl solutions is found to be isotonic with blood, while solute molecules in solution. It results in a change in the number
0.9% NaCl solution is isotonic with human RBCs. of particles in solution.

P
18 W JEE (XII) Module-1 CHEMISTRY
Association of solute particles Observed colligative property is proportional to observed number
of solute particle = (1 – α + nα).
The formation of a bigger molecule by the union of two, three
or more solute molecules is called association. Let ‘n’ simple 1 − α + nα
i=
molecules combine to give an associated molecule as: 1
non polar i = 1 + (n – 1)α

n( X ) ← →( X ) n
solvent
where α is the degree of dissociation of solute
As a result, the total number of particles in solution becomes less For strong electrolytes:
than the number of molecules initially dissolved in the solution and
if α = 1 or 100 % ⇒ then, i = n
hence the colligative properties will have lower value.
Example: NaCl, n = 2 ⇒ i = 2, for α = 100 %
As the molar mass of solute is inversely proportional to the
colligative properties, so the molar mass of solute will be greater K2SO4, n = 3 ⇒ i = 3, for α = 100 %
than theoretical value. Example: K4[Fe(CN)6], n = 5 ⇒ i = 5, for α = 100 %
Dissociation of solute molecules: For α = 50%, i = 1 + (n – 1) α = 3
Molecules of electrolytes undergo ionization or dissociation in Van’t Hoff factor and degree of association: The fraction of
ionizing solvents to give two or more particles in solution. For the total number of molecules of the substance which is present
example, AB ionizes in solution to give two particles. as associated molecules is known as the degree of association.
ionizing
→ A+ + B –
AB ← nA → [A]n
solvent
Initially 1 0
This dissociation results in an increase in the total number of α
After association (1 − α)
particles, and therefore the value of colligative properties of such n
solutions will be higher.
As the colligative properties are inversely related to molecular Total number of solute particles after association
weight, so the molecular weight of ionizable solute will be less  α
than the theoretical value.
1 − α +  mol
=
 n
Van’t Hoff factor (i) : Van’t Hoff in 1886 introduced a factor called
Observed C.P. ∝ observed number of solute particles
Van’t Hoff factor (i). The Van’t Hoff factor (i) is defined as
Number of solute particles after dissociation / association  α
i= 1 − α + 
=
Number of solute particles before dissociation / association  n
α
observed colligative property 1− α +
i= ,
i= n ⇒ i = 1− α + α
Normal colligative property n
1
Normal or actual molecular weight ( calculated ) α = Degree of association
i=
observed molecular mass ( experimental ) n = Number of solute particles which are associated.
In case of association of solute particles in solution, the observed 1
If α = 100% or 1 or α is not specified ⇒ i =
molecular weight of solute being more than the normal, the value n
of factor i, is less than unity, while for dissociation the value of i
is greater than unity because the observed molecular weight has Key Note
lesser value than normal molecular weight.
™ One mole of a solute like glucose, cane sugar, urea, etc.
Van’t Hoff factor and degree of dissociation: The fraction of when dissolved in 1000 g of water depresses the freezing
the total number of molecules which dissociates in the solution
point of water by 1.86ºC. However, solution of one
into simple ions or molecules is called the degree of dissociation.
mole of NaCl or BaCl2 in 1000 g of water depress the
A x By → xA y + + yBx − freezing point of water by 3.72ºC (2 × 1.86) and 5.58ºC
Initially 1 0 0 (3 × 1.86) respectively because each molecule of NaCl
After dissociation (1 − α) xα yα
and BaCl2 dissociates to give 2 (Na+ & Cl–) and three
(Ba2+ and 2Cl–) ions respectively and each ion behaves
Total number of solute particle = 1 – α + xα+ yα
as a particle in itself. Thus, among the 0.1 M solution
After dissociation = 1 – α + (x + y) α of the various substances like cane sugar, NaCl, BaCl2,
= (1 – α + nα) mol Al2(SO4)3, etc. the colligative effect will be maximum in
where n = x + y = total sum of no. of cations and anions given by Al2(SO4)3 solution giving 5 ions per molecule.
1 molecule of solute.

Solutions 19
™ The ideal solutions relations can be modified in
Similarly, there are certain substances which associate when
abnormal solution as given below:
dissolved in a particular solvent. Hence the molecular weight
of such substances as determined with the help of colligative ∆p
= i x2 , DTb = i Kb m
properties will be more than the theoretically calculated p10
value from the molecular formula. For example, the observed DTf = i kf m and p = i CRT
molecular weight of acetic acid when dissolved in benzene
™ The value of i is taken as one when solute is non
is found to be 118 which is nearly double the value (60) electrolyte e.g. glucose, sucrose, urea etc.
calculated from its molecular formula, CH3COOH. This is
™ All the 4 colligative properties are calculated by formula, only
due to the fact that acetic acid, in benzene, does not exist as for solution containing non-volatile solutes in a dilute solution.
CH3COOH but as a dimer, i.e. (CH3COOH)2.

ABNORMAL MOLECULAR MASSES


The molecular mass of a solute is inversely proportional to its
concentration or value of colligative properties. Example 22: A 5% solution of anhydrous CaCl2 at 0°C
developed 15 atm osmotic pressure. What is the degree of
(i) The extent of dissociation and colligative property: dissociation of CaCl2.
A solute which dissociates completely or partially in solution Sol. 5 g of CaCl2 is present in 100 mL, so 111 g (MW of
makes available more particles than would otherwise be present in 100 × 111
solution and therefore, a colligative property shows an increased CaCl2) will be present in = 2.22 litre
5 × 1000
effect. For example, molecular masses obtained of strong acids,
bases and salts are much less than their normal values. As an Now, π V = ST { n = 1}
example, one particle of potassium chloride on dissociation in 0.082 × 273 22.47
or π = = = 10.09 atm
water gives two particles, K+ and Cl– and therefore, the molecular 2.22 2.22
mass obtained by a colligative property is half of its normal We know that Van’t Hoff factor
molecular mass.
observed colligative property 15
K + Cl(s)

+ nH 2 O  +
→ K (aq) +
+ Cl(aq) i= =
Normal colligative property 10.09
1 mol 1 mol 1 mol
(ii) The extent of association and colligative property: i −1
CaCl2 → Ca2+ + 2Cl–, n = 3 and α = or
A solute that associates in solution provides less particles n −1
15
that would otherwise be present in solution and therefore, the −1
4.91
colligative property shows the decreased effect. For example, = α 10.09
=
benzoic acid in benzene is found to have molecular mass which is 3 −1 10.09 × 2
just twice its normal molecular mass. = 0.2433 or 24.33%
It is found that compounds which are capable of forming Example 23: Molal elevation constant for water is
0.52 K/m. 0.1 m solution of NaCl will boil at _____°C?
hydrogen bonds, e.g., phenols, carboxylic aids, alcohols: because
Sol. If van’t Hoff factor is i, then
of association show decreased effect of colligative property.
∆Tb = i. Kb. m (for NaCl, i = 2)
O H O ∆Tb = 2 × 0.52 × 0.1 = 0.104
H5C6 C6H5 ∆Tb = TbS – Tb° ⇒ 0.104 = TbS – 100
O H O
0.104 + 100 = TbS ⇒ 100.104 = TbS
A dimer of benzoic acid ∴ B. P. of solution is 100. 104°C
Key Note

Experimental value of colligative property


™ i=
Calculated value of this property when the solution behaves ideally
(∆Tb )obs (∆Tf )obs (∆p / p1º )obs πobs M cal
= = = 13. 19.5 g CH2FCOOH is dissolved in 500 g water. The
™ (∆Tb )cal (∆Tf )cal = (∆p / p1º )cal πobs M obs
depression in freezing point of water observed is 1.0ºC.
Where M is the molar mass of solute.
Calculate the van’t Hoff factor and dissociation constant
™ Knowing , the observed molar mass and the Van’t Hoff factor,
of CH2 FCOOH. kf(H2O) = 1.86 K kgmol– 1
i, the degree of dissociation, α can be easily calculated.

P
20 W JEE (XII) Module-1 CHEMISTRY
AARAMBH (SOLVED EXAMPLES)

1. If 20 mL of 0.5 M Na2SO4 is mixed with 50 mL of (c) 5.852 g/mL


0.2 M H2SO4 and 30 mL of 0.4 M Al2(SO4)3 solution. (d) 1.288 g/mL
Calculate [Na+], [H+], [Al3+], and [SO42–]. [Assuming  (JEE Lakshya Chemistry M-1)
100% dissociation]
Sol. Let V = 1 lt, then moles of solute = 6.9
 (JEE Lakshya Chemistry M-1)
wt. of solute = 6.9 × 56 gm
moles
Sol. Molarity = M solute
volume % by mass = × 100
M solution
(i) Na2SO4 → 2Na+ + SO42– 6.9 × 56
So, = × 100 = 30
10 m mol d × 1000
— 20 10 d = 1.288 g/mL
Total volume = 100 mL Therefore, option (d) is the correct answer.
20 4. The density of 3M solution of Na2S2O3 is 1.25 g mL–1.
∴ [Na+] = = 0.2 M
100 Calculate.
(ii) H2SO4 → 2H+ + SO42– (a) The % by weight of Na2S2O3
10 m mol (b) Mole fraction of Na2S2O3
— 20 10  (JEE Lakshya Chemistry M-1)
Total volume = 100 mL Sol. (a) mass of solution = 1000 × 1.25 = 1250 gm
20 mass of Na2S2O3 = 3 × 158 = 474
[H+] = = 0.2 M
100 474
% (w/w) = × 100 = 37.92
1250
(iii) Al2(SO4)3 → 2Al3+ + 3SO42–
Therefore, 37.92 is the correct answer.
12 m mol
— 24 36 3
(b) X Na 2S2O3 = = 0.065.
Total volume = 100 mL 3 + 43.11
1250 − 474
24 Mole of water solvent = = 43.11
[Al3+] = = 0.24 M 18
100
Therefore, 43.11 is the correct answer.
(iv) Total m mol of SO42– = m.m of SO42– from (Na2SO4 + H2SO4 5. Calculate molality of aqueous urea solution which has
+ Al2(SO4)3) = (10 + 10 + 36) = 56 Xurea = 0.2.
Total volume = 100 mL (a) 15.99 (b) 13.88
(c) 10.99 (d) 19.76
10 + 10 + 36 56
[SO42–] = = = 0.56 M  (JEE Lakshya Chemistry M-1)
100 100
Sol. Xu + Xw = 1
2. Calculate molality of 1.2 M H2SO4 solution. If its density
Xw = 1–0.2 = 0.8
(ρ) = 1.4 g/mL?
(a) 0.936 (b) 0.562 Let total moles = 1
(c) 0.386 (d) 0.425 nu = 0.2
 (JEE Lakshya Chemistry M-1) nw = 0.8
wwater = 0.8 × 18 = 14.4
1.2 × 1000
Sol. Molality = = 0.936 Number of moles of solute
1000 × 1.4 −1.2 × 98 Molality (m) =
Weight of solvent (kg)
Therefore, option (a) is the correct answer.
3. A 6.90 M solution of KOH in water has 30% by weight of 0.2
= = 13.88
KOH. Calculate density of solution (in g/mL). 14.4 / 1000
(a) 9.345 g/mL Therefore, option (b) is the correct answer.
(b) 2.445 g/mL

Solutions 21
6. 10 gm of a solute is dissolved in 80 gm of acetone. Vapour
pressure of this solution = 271 mm of Hg. If vapour pressure
of pure acetone is 283 mm of Hg, then calculate molar mass
of solute?
(a) 173 g/mol (b) 163 g/mol
(c) 143 g/mol (d) 183 g/mol
ice
 (JEE Lakshya Chemistry M-1)
P o – Ps w M
Sol. = × solute
Ps m W
(a) 900 gm (b) 600 gm
283-271 10 58 (c) 800 gm (d) 700 gm

⇒ = ×
271 m 80  (JEE Lakshya Chemistry M-1)
m = 163 g/mol. 0.1
Sol. ∆Tf = 0.2 = Kf × × 1000 …(i)
Therefore, option (b) is the correct answer. 1000
7. 1 L of aq. solution containing 60 g urea having density = 0.1
1.060 gm/mL is found to have ∆Tb = 0.5ºC. If temperature
∆Tf = 2 = Kf × × 1000 …(ii)
wt. of solvent
of this solution increases to 101.5ºC, then calculate amount
of water which must have gone in vapour state upto this on dividing eqn. (i) by (ii) we get,
point. [Given: Kb = 0.5 K kg mol–1 for water] wt. of solvent 0.2
= .
(a) 666.67 g (b) 999.97 g 1000 2
(c) 777.77 g (d) 888.87 g weight of remaining H2O is 100 gm and weight of ice
 (JEE Lakshya Chemistry M-1) formed is 900 gm.
Sol. Mass of solution = 1.060 × 103 = 1060 gm Therefore, option (a) is the correct answer.
mass of solvent = mass of solution – mass of solute
9. If 200 mL of 0.1 M urea solution is mixed with 300 mL of
w = 1060 – 60 = 1000 g 0.2 M glucose solution at 300 K, then calculate osmotic
∆Tb = i × Kb × m pressure due to solution.
n (a) 4.84 atm (b) 5.74 atm
m = × 1000 (c) 6.74 atm (d) 3.94 atm
W (g)
 (JEE Lakshya Chemistry M-1)
60 / 60
m = × 1000 C1V1 + C2 V2
1000 Sol. Cnet =
V1 + V2
m = 1
Now, π = Cnet RT

∆Tb = i × Kb × m

p = (0.04 + 0.12) × 0.0821 × 300 = 3.94 atm.
0.5 = i × 0.5 × 1
Therefore, option (d) is the correct answer.
i = 1
If temp. of solution increases, let w is wt. of water gone in 10. An aqueous solution of glucose boils at 100.01°C. The
vapour state molal elevation constant for water is 0.5 K mol–1 Kg. The
number of molecules of glucose in the solution containing
∆ Tb = TbS – Tb° = 101.5 – 100 = 1.5
100 g of water is

∆Tb = i × Kb × m (a) 6.022 × 1023 (b) 6.022 × 1022
60 (c) 12.046 × 1020 (d) 12.046 × 1023
1.5 = 1 × 0.5 × × 1000
60(1000 − w)  (JEE Lakshya Chemistry M-1)
⇒ Wwater = 666.67 g ∆Tb 0.01
Sol. ∆Tb = Kb.m Or m = = = 0.02 mole Kg–1 of water
Therefore, option (a) is the correct answer. Kb 0.5
8. 1000 gm H2O have 0.1 mole urea and its freezing point So, the number of moles of glucose in 100 g of water
is – 0.2ºC and now it is freezed upto – 2ºC, then how much 0.02 × 100
amount of ice (in g) will form? = = 0.002 moles of glucose
1000
= 0.002 × 6.022 × 1023 = 2 × 6.023 × 1020
Therefore, option (c) is the correct answer.

P
22 W JEE (XII) Module-1 CHEMISTRY
11. What is the percent by mass of iodine needed to reduce the (a) 23.14K kgmol– 1 (b) 30.14K kgmol– 1
freezing point of benzene to 3.5°C? The freezing point and
(c) 55.14K kgmol– 1 (d) 44.14K kgmol– 1
cryoscopic constant of pure benzene are 5.5°C and 5.12
K/m respectively.  (JEE Lakshya Chemistry M-1)
(a) 9.01% (b) 6.01% 20 / 65 20 × 1000
=
Sol. Molality of solid solution m =
(c) 8.01% (d) 7.01% 80 65 × 80
 (JEE Lakshya Chemistry M-1) 1000
Sol. ∆Tf = Tf0 – Tf = Kf . m
DTf = 1357 – 1268 = 89 = kf × m
5.5°C – 3.5°C = 5.12 × m 89 89 × 65 × 80
or, k=
f = = 23.14k kgmol− 1
2 m 20 × 1000
m= = 0.39 molal
5.12 Therefore, option (a) is the correct answer.

∴ Mass of iodine needed for 1000g of benzene 14. Molal elevation constant Kb for water is 0.52 K/m. 0.1 molal
= m × molecular mass of iodine I2 solution of NaCl will boil at
= 0.39 mol/kg × 254 g/mol = 99.06 g/kg (a) 100.52°C (b) 100.052°C

∴ 1000g + 99.06g solution contains 99.06g I2 (c) 101.04°C (d) 100.104°C
 (JEE Lakshya Chemistry M-1)
99.06g × 100
100g solution contains = 9.01% Sol. If vant Hoff factor is i, then
1099.06g
DTb = i Kb.m (for NaCl i = 2)
Therefore, option (a) is the correct answer.
= 2 × 0.52 × (0.1) = 0.104.
12. The B.P of chloroform was raised by 0.325 K when 0.5 gm of
And, so the boiling point of solution will be 100.104°C.
a compound was dissolved in 35 gm of chloroform. Calculate
the molar mass of the compound (Kb for chloroform = Therefore, option (d) is the correct answer.
3.9 K.kg/mole) 15. A solution is prepared by dissolving 26.3g of CdSO4 in
(a) 181.5 gm/mole (b) 171.4 gm/mole 1000g water. The depression in freezing point of solution
was found to be 0.284K. Calculate the Van’t Hoff factor. The
(c) 192.7 gm/mole (d) 199.6 gm/mole cryoscopic constant of water is 1.86K kg mol–1.
 (JEE Lakshya Chemistry M-1) (a) 1.21 (b) 3.41
(c) 4.77 (d) 7.44
0.5 / M 0.5 × 1000
Sol. Molality of Solution,
== m  (JEE Lakshya Chemistry M-1)
35 35 M
1000 Sol. Molecular mass (CdSO4) = 112.4 +32 + 4 × 16 = 208.4 g/mol
Where M is the molar mass of compound. Molality CdSO4
Mass CdSO 4
Now DTb = Kbm =
Molecular mass CdSO 4 × Mass solvent in kg
0.5 × 1000 26.3g
or, 0.325
= 3.9 × = 0.216m
35 M  1000 
(208.4g / mol) ×  kg 
or, M = 171.4 gm/mole  1000 
Therefore, option (b) is the correct answer. ∆Tf 0.284 K
DTf = i.Kf.m or i = = = 1.21
13. A brass sample composed of 20% Zinc and 80% copper by K f m 1.86K / m × 0.126m
mass melts at 1268K. Pure copper melts at 1357K. What is Therefore, option (a) is the correct answer.
the molal freezing point constant for copper?

Solutions 23
BOARD LEVEL PROBLEMS

SINGLE CORRECT TYPE QUESTIONS (c) Water will move from side (B) to side (A) if a pressure
equal to osmotic pressure is applied on piston (B).
1. Which of the following units is useful in relating concentration
(d) Water will move from side (A) to side (B) if pressure
of solution with its vapour pressure?
equal to osmotic pressure is applied on piston (A).
(a) Mole fraction (b) Parts per million
 (JEE Lakshya Chemistry M-1)
(c) Mass percentage (d) Molality
7. If two liquids A and B form minimum boiling azeotrope at
 (JEE Lakshya Chemistry M-1)
some specific composition then _______________.
2. The Van’t Hoff factor for 0.01 M Na2SO4 solution is 1.32.
(a) A–B interactions are stronger than those between A–A
The degree of dissociation is:
or B–B.
(a) 13% (b) 16%
(b) Vapour pressure of solution increases because more
(c) 20% (d) 18%
number of molecules of liquids A and B can escape
 (JEE Lakshya Chemistry M-1) from the solution.
3. Which of the following solutions would have the highest (c) Vapour pressure of solution decreases because less
osmotic pressure: number of molecules of only one of the liquids escape
M M from the solution.
(a) NaCl (b) Urea
10 10 (d) A–B interactions are weaker than those between A–A
or B–B.
M M
(c) BaCl2 (d) Glucose  (JEE Lakshya Chemistry M-1)
10 10
 (JEE Lakshya Chemistry M-1) 8. We have three aqueous solutions of NaCl labelled as ‘A’,
‘B’ and ‘C’ with concentrations 0.1M, 0.01M and 0.001M,
4. The freezing point of 11% aqueous solution of calcium nitrate respectively. The value of van’t Hoff factor for these
will be: solutions will be in the order______.
(a) 0°C (b) above 0°C
(a) i A < i B < i C (b) i A > iB > iC
(c) 1°C (d) below 0°C
(c) i A = i B = i C (d) i A < i B > i C
 (JEE Lakshya Chemistry M-1)
 (JEE Lakshya Chemistry M-1)
5. Henry’s law constant K of CO2 in water at 25°C is 3 × 10–2
mol/L atm–1. Calculation the mass of CO2 present in 100 L 9. 0.2 molal acid HX is 20 %. Ionised in solution. Kf = 1.86 K m–1.
of soft drink bottled with a partial pressure of CO2 of 4 atm The freezing point of the solution is
at the same temperature. (a) – 0.45°C (b) – 0.90°C
(a) 5.28 g (b) 12.0 g (c) – 0.31°C (d) – 0.53°C
(c) 428 g (d) 528 g
 (JEE Lakshya Chemistry M-1)
 (JEE Lakshya Chemistry M-1)
10. Which of the following statements is false?
6. Consider the following figure and mark the correct option.
(a) Two different solutions of sucrose of same molality
Piston (A) Piston (B)
prepared in different solvents will have the same
SPM depression in freezing point.
(b) The osmotic pressure of a solution is given by the
Concentrated equation Π = CRT ( where C is the molarity of the
sodium chloride solution).
Fresh water (A)
solution in (c) Decreasing order of osmotic pressure for 0.01 M aqueous
water (B) solutions of barium chloride, potassium chloride, acetic
acid and sucrose is BaCl2 > KCl > CH3COOH > sucrose.
(a) Water will move from side (A) to side (B) if a pressure (d) According to Raoult’s law, the vapour pressure exerted
lower than osmotic pressure is applied on piston (B). by a volatile component of a solution is directly
(b) Water will move from side (B) to side (A) if a pressure proportional to its mole fraction in the solution.
greater than osmotic pressure is applied on piston (B).
 (JEE Lakshya Chemistry M-1)

P
24 W JEE (XII) Module-1 CHEMISTRY
ASSERTION AND REASON TYPE QUESTIONS (a) (i)–(c), (ii)–(e), (iii)–(d), (iv)–(a), (v)–(b)
(b) (i)–(e), (ii)–(c), (iii)–(a), (iv)–(a), (v)–(d)
11. Assertion : When methyl alcohol is added to water, boiling
point of water increases. (c) (i)–(d), (ii)–(a), (iii)–(c), (iv)–(e), (v)–(b)
(d) (i)–(b), (ii)–(d), (iii)–(e), (iv)–(a), (v)–(c)
Reason : When a non-volatile solute is added to a volatile
solvent elevation in boiling point is observed.  (JEE Lakshya Chemistry M-1)
(a) Both Assertion and Reason are true and Reason is correct
explanation of Assertion. SHORT ANSWER TYPE QUESTIONS
(b) Both Assertion and Reason are true but Reason is not a
15. The vapour pressure of water is 12.3 Kpa at 300K. Calculate
correct explanation of Assertion.
vapour pressure of 1 molal solution of a non-volatile solute
(c) Assertion is true but Reason is false. in it. (JEE Lakshya Chemistry M-1)
(d) Assertion is false but Reason is true.
16. How many grams of KCl should be added to 1 kg of water
 (JEE Lakshya Chemistry M-1) to lower its freezing point to – 8.0ºC ? (Kf = 1.86 K kg/mol)
12. Assertion: When NaCl is added to water a depression in  (JEE Lakshya Chemistry M-1)
freezing point is observed.
17. An aqueous solution of 2% non-volatile solute exerts a
Reason: The lowering of vapour pressure of a solution causes pressure of 1.004 Bar at the normal boiling point of the
depression in the freezing point. solvent. What is the molar mass of the solute?
(a) Both Assertion and Reason are true and Reason is correct  (JEE Lakshya Chemistry M-1)
explanation of Assertion.
(b) Both Assertion and Reason are true but Reason is not a
correct explanation of Assertion.
LONG ANSWER TYPE QUESTIONS
(c) Assertion is true but Reason is false. 18. Equal moles of liquid P and Q are mixed. What is the ratio
(d) Assertion is false but Reason is true. of their moles in the vapour phase? Given that PP° = 2PQ°.

 (JEE Lakshya Chemistry M-1)  (JEE Lakshya Chemistry M-1)


19. Two liquids A and B boil at 145ºC and 190ºC respectively.
Which of them has higher vapour pressure at 80ºC ?
MATCH THE COLUMN TYPE QUESTIONS  (JEE Lakshya Chemistry M-1)
13. Match the column and choose correct option 20. (a) What is the value of ‘i’ for Al2(SO4)3 when it is completely
Vant’ Hoff factor Behaviour of compound dissociated? (b) Calculate the boiling point of a solution
prepared by adding 15.00 g of NaCl to 250 g of water.
(A) i=1 P. Impossible
(Kb = 0.512 K kg mol–1 and molar mass of NaCl = 58.44 g
(B) i>1 Q. Association is the solution mol–1)
(C) i<1 R. Dissociation in the solution  (JEE Lakshya Chemistry M-1)
21. (a) What is the value of ‘i’ when solute is associated and
(D) i=0 S. No dissociation or association
dissociated ?
(a) A–S, B–R, C–P, D–Q (b) Calculate the freezing point of an aqueous solution
(b) A–R, B–S, C–Q, D–P containing 10.50 g of MgBr2 in 200 g of water. (Molar
(c) A–S, B–P, C–R, D–Q mass of MgBr2 = 184, Kf = 1.86 K kg mol–1)
(d) A–S, B–R, C–Q, D–P  (JEE Lakshya Chemistry M-1)
 (JEE Lakshya Chemistry M-1) 22. (a) Outer shells of two eggs are removed. One of the egg is
placed in pure water and the other is placed in saturated
14. Match the law given in column I with expressions given in solution of NaCl. What will be observed and why?
column II.
(b) A solution prepared by dissolving 8.95 mg of a gene
Column I Column II fragment in 35.0 ml of water has an osmotic pressure of
(i) Raoult’s law (a) DTf = Kfm 0.335 ton at 25ºC. Assuming the gene fragment is a non-
electrolyte, determine the molar mass?
(ii) Henry’s law (b) Π = CRT
(iii) Elevation of (c) p = x1p1o + x2p2o  (JEE Lakshya Chemistry M-1)
boiling point 23. Explain the terms ideal and non-ideal solutions in the light of
(iv) Depression in (d) ∆Tb = Kbm forces of interactions operating between molecules in liquid
freezing point solutions?
(v) Osmotic pressure (e) p = KHx  (JEE Lakshya Chemistry M-1)

Solutions 25
24. Explain why on addition 1 mol of NaCl to 1 litre of water, Freezing point depression of different Sucrose solutions and
the boiling point of water increases, while addition of 1 mol coconut water.)
of methyl alcohol to one litre of water decreases its boiling
 (JEE Lakshya Chemistry M-1)
point?
I. When a non volatile solid is added to pure water it will:
 (JEE Lakshya Chemistry M-1)
(a) Boil above 100°C and freeze above 0°C
CASE STUDY BASED QUESTIONS (b) Boil below 100°C and freeze above 0°C
Read the passage given below and answer the following questions: (c) Boil below 100°C and freeze below 0°C
(d) Boil above 100°C and freeze below 0°C
25. Boiling point or freezing point of liquid solution would
be affected by the dissolved solids in the liquid phase. A II. Colligative properties are:
soluble solid in solution has the effect of raising its boiling (a) Dependent only on the concentration of the solute and
point and depressing its freezing point. The addition of non- independent of the solvent’s and solute’s identity.
volatile substances to a solvent decreases the vapor pressure (b) Dependent only on the identity of the solute and the
and the added solute particles affect the formation of pure concentration of the solute and independent of the
solvent crystals. According to many researches the decrease solvent’s identity.
in freezing point directly correlated to the concentration (c) Dependent on the identity of the solvent and solute and
of solutes dissolved in the solvent. This phenomenon is thus on the concentration of the solute.
expressed as freezing point depression and it is useful for (d) Dependent only on the identity of the solvent and the
several applications such as freeze concentration of liquid concentration of the solute and independent of the
food and to find the molar mass of an unknown solute in solute’s identity.
the solution. Freeze concentration is a high quality liquid III. Assume three samples of juices A, B and C have glucose as
food concentration method where water is removed by the only sugar present in them. The concentration of sample
forming ice crystals. This is done by cooling the liquid A, B and C are 0.1M, .5M and 0.2 M respectively. Freezing
food below the freezing point of the solution. The freezing point will be highest for the fruit juice:
point depression is referred as a colligative property and it is (a) A
proportional to the molar concentration of the solution (m), (b) B
along with vapour pressure lowering, boiling point elevation, (c) C
and osmotic pressure. These are physical characteristics of (d) All have same freezing point
solutions that depend only on the identity of the solvent IV. Identify which of the following is a colligative property:
and the concentration of the solute. The characters are not (a) Freezing point
depending on the solute’s identity. (Jayawardena, J. A. E. C., (b) Boiling point
Vanniarachchi, M. P. G. & Wansapala, M. A. J. (2017).
(c) Osmotic pressure
(d) All of the above

PRARAMBH (TOPICWISE)
3. 36 g of water and 828 g of ethyl alcohol form an ideal
solution. The mole fraction of water in it, is
CONCENTRATION TERMS (a) 1.0 (b) 0.7
(REVISION OF MOLE) (c) 0.4 (d) 0.1
1. The amount of anhydrous Na2CO3 present in 250 mL of  (JEE Lakshya Chemistry M-1)
0.25 M solution is
4. An X molal solution of a compound in benzene has mole
(a) 225 g (b) 66.25 g fraction of solute equal to 0.2. The value of X is
(c) 6.0 g (d) 6.625 g (a) 14 (b) 3.2 (c) 4 (d) 2
 (JEE Lakshya Chemistry M-1)  (JEE Lakshya Chemistry M-1)
2. 2.0 molar solution is obtained, when 0.5 mole solute is 5. 4.0 gm of NaOH is contained in one decilitre of solution. Its
dissolved in molarity would be
(a) 250 mL solvent (b) 250 g solvent (a) 4 M (b) 2 M
(c) 250 mL solution (d) 1000 mL solvent (c) 1 M (d) 1.5 M
 (JEE Lakshya Chemistry M-1)  (JEE Lakshya Chemistry M-1)

P
26 W JEE (XII) Module-1 CHEMISTRY
HENRY’S LAW AND BASICS OF VAPOUR 13. Two liquids A and B form an ideal solution, What is the
vapour pressure of solution containing 2 moles of A and
PRESSURE 3 moles of B at 300 K? [Given: At 300 K, Vapour pressure
6. Calculate the solubility of gaseous oxygen in water at a of pure liquid A (PºA ) = 100 torr, Vapour pressure of pure
temperature of 293 K when the partial pressure exerted by liquid B (PºB ) = 300 torr]
O2 is 1 bar. (Given: KH for O2 34840 bar L mol–1)
(a) 2.87 × 10–5 mol/L (b) 4 × 10–4 mol/L (a) 200 torr (b) 140 torr
(c) 180 torr (d) 220 torr
(c) 2.87 × 10–4 mol/L (d) 5 × 10–4 mol/L
 (JEE Lakshya Chemistry M-1)
 (JEE Lakshya Chemistry M-1)
14. The vapour pressure of pure benzene and toluene are 160
7. Solubility of gas in water is x g/cm3 at 300 K temperature.
and 60 torr respectively. The mole fraction of toluene in
When temperature increases to 400 K, then solubility of gas
vapour phase in contact with equimolar solution of benzene
(a) Increases (b) Decreases and toluene is
1 (a) 0.50 (b) 0.60 (c) 0.27 (d) 0.73
(c) Remain same (d) x
2  (JEE Lakshya Chemistry M-1)
 (JEE Lakshya Chemistry M-1)
DIFFERENCE BETWEEN IDEAL AND
8. Which one of the following gases has the lowest value of
Henry’s law constant? NON-IDEAL SOLUTION
(a) N2 (b) He 15. Which pair from the following will not form an ideal
solution?
(c) H2 (d) CO2
(a) CCl4 + SiCl4 (b) H2O + C4H9OH
 (JEE Lakshya Chemistry M-1)
(c) C2H5Br + C2H5l (d) C6H14 + C7H16
9. The vapour pressure of water depends upon
 (JEE Lakshya Chemistry M-1)
(a) Surface area of container
16. An ideal solution is that which
(b) Volume of container
(a) Shows positive deviation from Raoult’s law
(c) Temperature
(b) Shows negative deviation from Raoult’s law
(d) All of these (c) Has no connection with Raoult’s law
 (JEE Lakshya Chemistry M-1) (d) Obeys Raoult’s law
10. A vessel has nitrogen gas and water vapours in equilibrium  (JEE Lakshya Chemistry M-1)
with liquid water at a total pressure of 1 atm. The partial 17. Which property is shown by an ideal solution?
pressure of water vapours is 0.3 atm. The volume of this (a) It follows Raoult’s law (b) ∆Hmix = 0
vessel is reduced to one third of the original volume, at the (c) ∆Vmix = 0 (d) All of these
same temperature, then total pressure of the system is
 (JEE Lakshya Chemistry M-1)
(Neglect volume occupied by liquid water)
18. When two liquids A and B are mixed then their boiling
(a) 3.0 atm (b) 1 atm
points becomes greater than both of them. What is the
(c) 3.33 atm (d) 2.4 atm nature of this solution?
 (JEE Lakshya Chemistry M-1) (a) Ideal solution
(b) Non-ideal solution with positive deviation
V.P OF MIXTURE OF LIQUIDS (c) Non-ideal solution with negative deviation
11. For a solution of two liquids A and B it was proved that (d) Normal solution
P = XA ( PA0 − P B0 ) + PB0 . The solution is
 (JEE Lakshya Chemistry M-1)
(a) Ideal (b) Non-ideal 19. All form ideal solution except
(c) Semi ideal (d) None of these (a) C2H5Br and C2H5l
 (JEE Lakshya Chemistry M-1) (b) C6H5Cl and C6H5Br
(c) C6H6 and C6 H5CH3
12. 1 mole of n-heptane (V.P. = 92 mm of Hg) was mixed
with 4 moles of n-octane (V.P = 31 mm of Hg), the vapour (d) C2H5l and C2H5OH
pressure of the resulting ideal solution is  (JEE Lakshya Chemistry M-1)
(a) 46.2 mm of Hg (b) 40.0 mm of Hg 20. An azeotropic solution of two liquids has boiling point
(c) 43.2 mm of Hg (d) 38.4 mm of Hg lower than either when it
 (JEE Lakshya Chemistry M-1) (a) Shows a negative deviation from Raoult’s law

Solutions 27
(b) Shows no deviation from Raoult’s law 27. If 0.15 g of a non-volatile solute dissolved in 15 g of solvent
(c) Shows positive deviation from Raoult’s law has boiling point higher by 0.216°C than that of the pure
(d) Is saturated solvent, then molecular weight (in g/mol) of the substance
(molal elevation constant for the solvent is 2.16° K kg/mol) is
 (JEE Lakshya Chemistry M-1)
(a) 1.01 (b) 10 (c) 10.1 (d) 100
 (JEE Lakshya Chemistry M-1)
RELATIVE LOWERING IN V.P
28. Mark the correct relationship between the boiling points of
21. For a solution of volatile liquids, the partial vapour pressure
very dilute solutions of BaCl2, (T1) and KCI (T2), having
of each component in solution is directly proportional to
the same molarity
(a) Molarity (b) Mole fraction (a) T1 = T2
(c) Molality (d) Normality (b) T1 > T2
 (JEE Lakshya Chemistry M-1) (c) T2 > T1
22. When a substance is dissolved in a solvent, the vapour (d) T2 is approximately equal to T1
pressure of the solvent is decreased. This results in  (JEE Lakshya Chemistry M-1)
(a) An increase in the b.p. of the solution 29. The molal freezing point constant for water is 1.86°C
(b) A decrease in the b.p. of the solvent kg mole–1. If 342 gm of canesugar (C12H22O11) are dissolved
(c) The solution having a higher freezing point than the solvent in 1000 gm of water, the solution will freeze at
(d) The solution having a lower osmotic pressure than the (a) –1.86°C (b) 1.86°C
solvent (c) – 3.92°C (d) 2.42°C
 (JEE Lakshya Chemistry M-1)  (JEE Lakshya Chemistry M-1)
23. The vapour pressure lowering caused by the addition of 100 g 30. The freezing point of one molal NaCl solution assuming
of sucrose (molecular mass = 342) to 1000 g of water if the NaCl to be 100 % dissociated in water is (molal freezing
vapour pressure of pure water at 25ºC is 23.8 mm Hg, is point depression constant = 1.86 K kg mol–1)
(a) – 1.86°C (b) – 3.72°C
(a) 1.25 mm Hg (b) 0.125 mm Hg
(c) + 1.86°C (d) + 3.72°C
(c) 1.15 mm Hg (d) 00.12 mm Hg
 (JEE Lakshya Chemistry M-1)
 (JEE Lakshya Chemistry M-1)
31. 1.00 gm of a non-electrolyte solute when dissolved in 50 gm
24. According to Raoult’s law, the relative lowering of vapour of benzene lowers the freezing point of benzene by 0.40 K.
pressure of a solution of non-volatile substance is equal to Kf for benzene is 5.12 kg mol–1. Molecular mass of the
(a) Mole fraction of the solvent solute will be
(b) Mole fraction of the solute (a) 256 g mol–1 (b) 2.56 g mol–1
(c) Weight percentage of a solute (c) 512×103 g mol–1 (d) 2.56×104 g mol–1
(d) Weight percentage of a solvent  (JEE Lakshya Chemistry M-1)
 (JEE Lakshya Chemistry M-1)
25. The vapour pressure of water at 20°C is 17.54 mm Hg. OSMOTIC PRESSURE
When 20 g of a non-ionic, substance is dissolved in 100 g 32. The concentration in gms per litre of a solution of cane sugar
of water, the vapour pressure is lowered by 0.30 mm Hg. (M = 342) which is isotonic with a solution containing 6
What is the molecular weight (in g/mol) of the substance? gms of urea (M = 60) per litre is
(Do not consider solution as dilute) (a) 3.42 (b) 34.2
(a) 210.2 (b) 206.88 (c) 5.7 (d) 19
(c) 215.2 (d) 200.8
 (JEE Lakshya Chemistry M-1)
 (JEE Lakshya Chemistry M-1)
33. Osmotic pressure is 0.0821 atm at temperature of 300 K.
Find concentration in mole/litre
ELEVATION OF BOILING POINT AND
(a) 0.033 (b) 0.066
DEPRESSION OF FREEZING POINT (c) 0.33 × 10–2 (d) 0.3
26. The latent heat of vapourisation of water is 9700 Cal/mole
and if the b.p. is 100° C, ebullioscopic constant of water is  (JEE Lakshya Chemistry M-1)
(a) 0.513°C/molal (b) 1.026°C/molal 34. A 5 % solution of cane sugar (mol. wt. = 342) is isotonic
(c) 10.26°C/molal (d) 1.832°C/molal with 1 % solution of a substance X. The molecular weight
(in g/mol) of X is
 (JEE Lakshya Chemistry M-1)

P
28 W JEE (XII) Module-1 CHEMISTRY
(a) 34.2 (b) 171.2 (a) 0.1 M NaCl
(c) 68.4 (d) 136.8 (b) 0.1 Na2SO4
 (JEE Lakshya Chemistry M-1) (c) 0.1 M BaCl2
35. Which statement is wrong regarding osmotic pressure (p), (d) 0.1 M glucose
volume (V) and temperature (T)?  (JEE Lakshya Chemistry M-1)
1 41. Which of the following aqueous solutions containing 10 gm
(a) π ∝ , if T is constant
V of solute in each case has highest B.P.?
(b) p ∝ T, if V is constant (a) NaCl solution
(c) p ∝ V, if T is constant (b) KCl solution
(d) pV is constant, if T is constant (c) Sugar solution
(d) Glucose solution
 (JEE Lakshya Chemistry M-1)
 (JEE Lakshya Chemistry M-1)
36. The value of osmotic pressure of a 0.2 M aqueous solution
at 293 K is 42. The freezing points of equimolar solutions of glucose,
(a) 8.4 atm (b) 0.48 atm KNO3 and AlCl3 are in the order of
(c) 4.8 atm (d) 4.0 atm (a) AlCl3 < KNO3 < Glucose
(b) Glucose < KNO3 < AlCl3
 (JEE Lakshya Chemistry M-1)
(c) Glucose < AlCl3 < KNO3
37. If a 0.1 M solution of glucose (mol. wt. 180) and 0.1 molar (d) AlCl3 <Glucose < KNO3
solution of urea (mol. wt. 60) are placed on the two sides of
 (JEE Lakshya Chemistry M-1)
a semipermeable membrane to equal heights, then it will be
43. Which of the following salt has the same value of Van’t
correct to say that
Hoff factor (i) as that of K4 [Fe(CN)6]?
(a) There will be no net movement across the membrane.
(a) Al2(SO4)3 (b) NaCl
(b) Glucose will move across the membrane into urea (c) Na2SO4 (d) Al(NO3)3
solution.
 (JEE Lakshya Chemistry M-1)
(c) Urea will flow across the membrane into glucose
44. One mole of a solute A is dissolved in a given volume of a
solution. solvent. The association of the solute take place according
(d) Water will flow from urea solution into glucose to nA  An.
solution. If x is the degree of association of A, then Van’t Hoff factor
 (JEE Lakshya Chemistry M-1) (i) is expressed as
38. A solution of urea contain 8.6 gm/litre (mol. wt. 60.0). x
(a) i =1 – x (b) i = 1 +
It is isotonic with a 5% solution of a non-volatile solute. n
The molecular weight (in g/mol) of the solute will be x
1− x +
(a) 348.9 (b) 34.89 (c) i = n (d) i = 1
1
(c) 3489 (d) 861.2
 (JEE Lakshya Chemistry M-1)
 (JEE Lakshya Chemistry M-1)
45. The molecular weight of benzoic acid in benzene as determined
39. Osmotic pressure of 0.1 M solution of NaCl and Na2SO4 by depression in freezing point method corresponds to
will be
(a) Ionization of benzoic acid
(a) Same (b) Dimerization of benzoic acid
(b) Osmotic pressure of NaCl solution will be more than (c) Trimerization of benzoic acid
Na2SO4 solution. (d) Solvation of benzoic acid
(c) Osmotic pressure of Na2SO4 solution will be more than  (JEE Lakshya Chemistry M-1)
NaCl.
46. What is the freezing point of a solution containing 8.1 g
(d) Osmotic pressure of Na2SO4 will be less than that of HBr in 100 g water assuming the acid to be 90 % ionised?
NaCl solution. (Kf for water = 1.86 K mole–1)
 (JEE Lakshya Chemistry M-1) (a) 0.85°C (b) – 3.53°C (c) 0°C (d) – 0.35°C
 (JEE Lakshya Chemistry M-1)
VAN’T HOFF FACTOR
40. Which of the following will have the highest freezing point
at 1 atm pressure?

Solutions 29
PRABAL (JEE MAIN LEVEL)

1. Mole fraction of C3H5(OH)3 (glycerine) in a solution of (non-volatile) are added to this solution, the vapour pressure
36 g of water and 46 g of glycerine is of solution increases by 30 torr. What is the value of ?
(a) 0.46 (b) 0.36 (a) 940 torr (b) 405 torr
(c) 0.20 (d) 0.40 (c) 90 torr (d) None of these
 (JEE Lakshya Chemistry M-1)  (JEE Lakshya Chemistry M-1)
2. An aqueous solution of urea containing 18 g urea in 9. At a constant temperature, ∆S will be maximum for which
1500 cm3 of solution has a density of 1.052 g/cm3.
of the following processes?
If the molecular weight of urea is 60, then the molality of
solution is (a) Vaporisation of a pure solvent
(a) 0.2 (b) 0.192 (b) Vaporisation of solvent from a solution containing
non-volatile and non electrolyte solute in it.
(b) 0.064 (d) 1.2
 (JEE Lakshya Chemistry M-1) (c) Vaporisation of solvent from a solution containing
non-volatile but electrolyte solute in it.
3. What is the molarity of H2SO4 solution that has a density of
1.84 g/cc at 35°C and contains 98% by weight? (d) Entropy change will be same in all the above cases.
(a) 4.18 M (b) 8.14 M  (JEE Lakshya Chemistry M-1)
(c) 18.4 M (d) 18 M 10. Which of the following is less than zero for ideal solutions?
(a) ∆Hmix (b) ∆Gmix (c) ∆Vmix (d) ∆Smix
 (JEE Lakshya Chemistry M-1)
4. When 5.0 gram of BaCl2 is dissolved in water to have 106  (JEE Lakshya Chemistry M-1)
gram of solution. The concentration of solution is
11. The vapour pressures of pure liquids ‘A’ and ‘B’ are 300
(a) 2.5 ppm (b) 5 ppm and 800 torr respectively at 25°C. When these two liquids
(c) 5 M (d) 5 gm L–1 are mixed at this temperature to form a solution in which
mole percentage of “B” is 92, then the total vapour pressure
 (JEE Lakshya Chemistry M-1)
is observed to be 0.95 atm. Which of the following is true
5. A mixture contains 1 mole of volatile liquid A for this solution?
( PAO = 100 Hg ) and 3 moles of volatile liquid B (a) ∆Vmix > 0 (b) ∆Hmix < 0
(c) ∆Vmix = 0 (d) ∆Hmix = 0
(P O
B= 80 mm Hg ) . If solution behaves ideally, the total
vapour pressure of the distillate is  (JEE Lakshya Chemistry M-1)
(a) 80 mm Hg (b) 85.88 mm Hg 12. Consider a binary mixture of volatile liquids. If at
(c) 90 mm Hg (d) 92 mm Hg XA = 0.4, the vapour pressure of solution is 580 torr then the
O O
 (JEE Lakshya Chemistry M-1) mixture could be [ PA = 300 torr, PB = 800 torr]
(a) H2O – HNO3 (b) C6H5Cl – C6H5Br
6. Mixture of volatile components A and B has total vapour
(c) C6H6 – C6H5CH3 (d) nC6H14 – nC7H16
pressure (in Torr) P = 254 –119 XA where XA is mole
 (JEE Lakshya Chemistry M-1)
fraction of A in mixture. Hence, PAO and PBO are (in Torr)
13. Which of the following mixture of liquids will form an
(a) 254, 119 (b) 119, 254 ideal solution?
(c) 135, 254 (d) 119, 373 (a) C2H5OH and water (b) HNO3 and water
 (JEE Lakshya Chemistry M-1) (c) CHCl3 and CH3COCH3(d) C6H6 and C6H5CH3
7. What is ratio of mole fraction of benzene (PºB = 150 torr)  (JEE Lakshya Chemistry M-1)
and toluene (PºT = 50 torr) in vapour phase if the given 14. Which of the following mixture shows negative deviation
solution has a vapour pressure of 120 torr? from Raoult’s law?
(a) 7 : 1 (b) 7 : 3 (a) CHCl3 and acetone (b) CHCl3 and C2H5OH
(c) 8 : 1 (d) 7 : 8 (c) C6H5CH3 and C6H6 (d) C6H6 and CCl4
 (JEE Lakshya Chemistry M-1)  (JEE Lakshya Chemistry M-1)
8. At 300 K. the vapour pressure of an ideal solution 15. Assuming each salt to be completely dissociated, which of
containing 3 mole of A and 2 mole of B is 600 torr. the following will have the highest osmotic pressure?
At the same temperature, if 1.5 mole of A and 0.5 mole of C (a) Decimolar Al2(SO4)3

P
30 W JEE (XII) Module-1 CHEMISTRY
(b) Decimolar BaCl2 24. 1 mol each of following solutes are taken in 5 mol water.
(c) Decimolar Na2SO4 (A) NaCl (B) K2SO4
(d) A solution obtained by mixing equal volumes of (C) Na3PO4 (D) Glucose
(b) and (c) and filtering Assuming 100% ionisation of each electrolyte, vapour
 (JEE Lakshya Chemistry M-1) pressure of solutions will be in the order
16. A complex containing K+,
Pt(IV) and Cl– is 100% ionised (a) D < A < B < C (b) D < C < B < A
giving i = 3. Thus, complex is (c) C < B < A < D (d) Equal for all
(a) K2 [PtCl4] (b) K2[PtCl6]  (JEE Lakshya Chemistry M-1)
(c) K3[PtCl5] (d) K[PtCl3] 25. The vapour pressure of a solvent is decreased by 10 mm
 (JEE Lakshya Chemistry M-1) of Hgwhen a non-volatile solute was added to the pure
solvent. The mole fraction of solute in solution is 0.2, what
17. pH of 1 M HA (weak acid) is 2. Hence, van’t Hoff factor of
would be mole fraction of the solvent if decrease in vapour
HA is
pressure is 20 mm of Hg?
(a) 1.2 (b) 1.02
(a) 0.2 (b) 0.4 (c) 0.6 (d) 0.8
(c) 1.1 (d) 1.01
 (JEE Lakshya Chemistry M-1)
 (JEE Lakshya Chemistry M-1)
26. The vapour pressure of a saturated solution of sparingly
18. In which case, van’t Hoff factor is maximum? soluble salt (XCl3) was 17.20 mm Hg at 27°C. If the vapour
(a) KCl, 50% ionised (b) K2SO4 40% ionised pressure of pure H2O is 17.25 mm Hg at 300 K. What is the
(c) FeCl3 30% ionised (d) SnCl4 20% ionised solubility of sparingly soluble salt XCl3 in mole/Litre?
 (JEE Lakshya Chemistry M-1) (a) 4.04 × 10–2 (b) 8.08 × 10–2
19. The vapour pressure of water at room temperature is lowered (c) 2.02 × 10–2 (d) 4.04 × 10–3
by 5% by dissolving a solute in it, then the approximate  (JEE Lakshya Chemistry M-1)
molality of solution is 27. 0.5 M, 100 mL A and 0.2 M, 500 mL B are mixed at 27°C.
(a) 2 (b) 1 (c) 4 (d) 3 Vapour pressure of pure A and pure B is 200 mm Hg and
 (JEE Lakshya Chemistry M-1) 50 mm Hg respectively at 27°C. The ratio of partial pressures
of A and B (in vapour phase) after mixing is
20. The van’t Hoff factor for 0.1 M Ba(NO3)2 solution is 2.74.
(a) 2 : 1 (b) 1 : 2 (c) 2 : 3 (d) 4 : 1
The degree of dissociation is
(a) 91.3% (b) 87% (c) 100% (d) 74%  (JEE Lakshya Chemistry M-1)
 (JEE Lakshya Chemistry M-1) 28. Which of the solutions have equal elevation in boiling point?
A. 0.1 M Na2SO4
21. The vapour pressure of pure liquid A is 10 torr and at the
same temperature when 1 g of B solid is dissolved in 20 g of B. 0.1 M MgCl2
A, its vapour pressure is reduced to 9.0 torr. If the molecular C. 0.1 M C6H12O6 (glucose)
mass of A is 200 amu, then the molecular mass of B is D. 0.1 M Al(NO3)3
(a) 100 amu (b) 90 amu (a) A and B (b) B and C
(c) 75 amu (d) 120 amu (c) C and D (d) None of these
 (JEE Lakshya Chemistry M-1)  (JEE Lakshya Chemistry M-1)
22. The vapour pressure of a solution of a non-volatile solute 29. Aluminium phosphate is 100% ionised in 0.01 molal aqueous
B in a solvent A is 95% of the vapour pressure of the pure solution. Hence, ∆Tb/Kb is
solvent at the same temperature. If the molecular weight of (a) 0.01 (b) 0.015
the solvent is 0.3 times the molecular weight of the solute, (c) 0.0175 (d) 0.02
what is the ratio of weight of solvent to solute?
 (JEE Lakshya Chemistry M-1)
(a) 0.15 (b) 5.7
30. A 0.001 molal solution of a complex MA8 in water has the
(c) 0.2 (d) None of these freezing point of – 0.0054°C. Assuming 100% ionization of
 (JEE Lakshya Chemistry M-1) the complex salt and Kr for H2O = 1.86 K m–1, what is the
23. The vapour pressure of a dilute aqueous solution of glucose is correct representation for the complex?
750 mm of mercury at 373 K. The mole fraction of solute is (a) [MA5] (b) [MA7]A
1 1 1 1 (c) [MA6]A2 (d) [MA5]A3
(a) (b) (c) (d)
10 7.6 35 76  (JEE Lakshya Chemistry M-1)
 (JEE Lakshya Chemistry M-1)

Solutions 31
31. Which of the following option has been arranged in the order 38. A solution of a substance containing 1.05 g per 100 mL was
of decreasing freezing point? found to be isotonic with 3% by mass of glucose solution.
(a) 0.05 M KNO3 > 0.04 M BaCl2 > 0.140 M sucrose > The molecular mass (in g/mol) of the substance is
0.075 M CuSO4 (a) 31.5 (b) 6.3 (c) 630 (d) 63
(b) 0.04 M BaCl2 > 0.140 M sucrose > 0.075 M CuSO4 >  (JEE Lakshya Chemistry M-1)
0.05 M KNO3
39. The relationship between osmotic pressure at 273 K when
(c) 0.075 M CuSO4 > 0.140 M sucrose > 0.04 M BaCl2 >
0.05 M KNO3 10 g glucose (P1), 10 g urea (P2) and 10 g sucrose (P3) are
dissolved in 250 mL of water is
(d) 0.075 M CuSO4 > 0.05 M KNO3 > 0.140 M sucrose >
0.04 M BaCl2 (a) P1 > P2 > P3 (b) P3 > P1 > P2
(c) P2 > P1 > P3 (d) P2 > P3 > P1
 (JEE Lakshya Chemistry M-1)
 (JEE Lakshya Chemistry M-1)
32. A 0.2 molal aqueous solution of a weak acid (HX)
is 20% ionised. The freezing point of this solution is 40. Which one of the following pairs of solution can we expect
(Given: Kf = 1.86°C kg mol–1 for water) to be isotonic at the same temperature?
(a) – 0.45°C (b) – 0.90°C (a) 0.1 M urea and 0.1 M NaCl
(c) – 0.31°C (d) – 0.53°C (b) 0.1 M urea and 0.2 M MgCl2
(c) 0.1 M NaCl and 0.1 M Na2SO4
 (JEE Lakshya Chemistry M-1)
(d) 0.1 M Ca(NO3)2 and 0.1 M Na2SO4
33. A complex of iron and cyanide ions is 100% ionised at 1 m
(molal). If its elevation in b.p. is 2.08, then the complex is  (JEE Lakshya Chemistry M-1)
(Kb = 0.52°C mol–1 kg) 41. FeCl3 on reaction with K4[Fe(CN6)] in aqueous solution
(a) K3[Fe(CN)6] (b) Fe(CN)2 gives blue colour. These two solutions are separated by a
semipermeable membrane AB as shown. Due to osmosis,
(c) K4[Fe(CN)6] (d) Fe(CN)4
there is
 (JEE Lakshya Chemistry M-1) 0.01 M 0.01 M
34. A solution of x moles of sucrose in 100 grams of water freezes K4Fe(CN)6 FeCl3
at –0.2°C. As ice separates, the freezing point goes down to
–0.25°C. How many grams of ice would have separated?
(a) 18 grams (b) 20 grams Side X Side Y
(c) 25 grams (d) 23 grams
 (JEE Lakshya Chemistry M-1) SPM

35. Elevation of boiling point of 1 molar aqueous glucose (a)


Blue colour formation on side X
solution (density = 1.2 g/mL) is (b)
Blue colour formation on side Y
(a) Kb (b) 1.20 Kb (c)
Blue colour formation on both sides of X and Y
(c) 1.02 Kb (d) 0.98 Kb (d)
No blue colour formation
 (JEE Lakshya Chemistry M-1)
 (JEE Lakshya Chemistry M-1)
42. The partial pressure of ethane over a solution containing
36. Osmotic pressure of blood is 7.40 atm at 27°C. Number of 6.56 × 10–2 g of ethane is 1 bar. If the solution contains
mol of glucose to be used per L for an intravenous injection 5.00 × 10–2 g ethane, then partial pressure is ______
that is to have the same osmotic pressure as blood, is
(a) 1.312 bar (b) 1.0 bar
(a) 0.3 (b) 0.2
(c) 0.762 bar (d) 0.912 bar
(c) 0.1 (d) 0.4
 (JEE Lakshya Chemistry M-1)
 (JEE Lakshya Chemistry M-1)
43. Two elements A and B form compounds AB2 and AB4 (both are
37. A solution of glucose (C6H12O6) is isotonic with 4 g of urea non-volatile and non-electrolyte). When dissolved in 20.0 g of
(NH2–CO–NH2) per liter of solution. The concentration of benzene, 1 g of AB2 lowers the freezing point of benzene by 2.3
glucose is
K, whereas 1 .0 g of AB4 lowers it by 1 .3 K [Kf (benzene) = 5.52
(a) 4 g/l (b) 8 g/l K mol–1 kg]. Thus, atomic masses of A and B respectively, are
(c) 12 g/l (d) 14 g/l
(a) 120, 212 (b) 96, 28
 (JEE Lakshya Chemistry M-1)
(c) 212, 120 (d) 28, 46
 (JEE Lakshya Chemistry M-1)

P
32 W JEE (XII) Module-1 CHEMISTRY
44. Liquids A and B form an ideal solution. At 30° C, the total 0.25 molal solution. The degree of association of solute
vapour pressure of a solution containing 1 mol of A and ‘A’ is found to be 0.80. The freezing point of benzene is
2 moles of B is 250 mm Hg. The total vapour pressure 5.5°C and its cryoscopic constant is 5.12 K m–1.
becomes 300 mm Hg when 1 more mol of A is added to the  (JEE Lakshya Chemistry M-1)
first solution. The vapour pressures (in mm Hg) of pure A 51. Two liquids A and B have PA0 : PB0 = 1 : 3 at a certain
and B at the same temperature are temperature. If the mole fraction ratio of XA : XB = 1:3,
the mole percentage of A in vapour in equilibrium with
(a) 450, 150 (b) 250, 300 the solution at a given temperature is _________.
(c) 125, 150 (d) 150, 450  (JEE Lakshya Chemistry M-1)
 (JEE Lakshya Chemistry M-1) 52. 500 mL of 6% (w/V) urea solution is mixed with 500 mL of
18 % (w/V) glucose solution and final volume of mixture
45. If sodium sulphate is considered to be completely dissociated
becomes 2 L after addition of water. The osmotic pressure
into cations and anions in aqueous solution, the change in of final 2 L solution at 27°C is 5yR. The value of y is
freezing point of water(∆ Tf),when 0.01 mol of sodium sulphate [R → universal gas constant]
is dissolved in 1 kg of water, is (Kf = 1.86 K kg mol – 1)  (JEE Lakshya Chemistry M-1)
(a) 0.0186 K (b) 0.0372 K 53. The molal freezing point depression constant of benzene
(c) 0.0558 K (d) 0.0744 K (C6H6) is 4.90 K kg mol– 1. Selenium exists as a polymer
of the type SeX. When 3.26 g of selenium is dissolved in
 (JEE Lakshya Chemistry M-1) 226 g of benzene, the observed freezing point is 0.112°C
46. The vapour pressure of acetone at 20° C is 185 torr. When lower than that of pure benzene. What is the value of X.
(Atomic mass of Se = 78.8 mol– 1).
1.2 g of non-volatile substance was dissolved in 100 g of
acetone at 20° C, its vapour pressure was 183 torr. The molar  (JEE Lakshya Chemistry M-1)
mass (g mol– 1) of the substance is 54. A radiator was filled with 10L of water to which 2.5 L of
(a) 128 (b) 448 (c) 32 (d) 64 methanol (density = 0.8g.ml–1) were added. The vehicle
is parked outdoors where the temperature is 0°C. The
 (JEE Lakshya Chemistry M-1) temperature is decreasing at a uniform rate of 0.50C/min.
47. For 1 molal aqueous solution of the following compounds, Upto what time (in min) will there be no danger to the
which one will show the highest freezing point? radiator of the car? Kf(water) = 1.86 kg.mol–1.K. Assume
methanol to be non-volatile. Multiply final answer by four.
(a) [Co(H2O)5Cl]Cl2·H2O (b) [Co(H2O)4Cl2]Cl·2H2O
 (JEE Lakshya Chemistry M-1)
(c) [Co(H2O)3Cl3]·3H2O (d) [Co(H2O)6]Cl3
55. 24.6 litre dry N2 gas is slowly passed over a liquid at 27°C
 (JEE Lakshya Chemistry M-1) and 1 atm. The pressure of gaseous sample coming out
48. A solution at 20°C is composed of 1.5 mol of benzene and increases due to saturation of gas with vapours of the liquid.
3.5 mol of toluene. If the vapour pressure of pure benzene and If the mass ratio of N2 gas and the liquid vapour in the
pure toluene at this temperature are 74.7 torr and 22.3 torr, gaseous sample coming out is 25 : 4 and the vapour pressure
respectively, then the total vapour pressure of the solution of the liquid at 27°C is 0.04 atm, the molar mass (gm/mol)
and the benzene mole fraction in equilibrium (in vapor phase) of liquid is.
with it will be, respectively  (JEE Lakshya Chemistry M-1)
(a) 38.0 torr and 0.589 (b) 30.5 torr and 0.389 56. H2S, a toxic gas with rotten egg like smell, is used for the
(c) 35.8 torr and 0.280 (d) 35.0 torr and 0.480 quantitative analysis. If the solubility of H2S in water at STP
is 0.2m, Henry’s law constant (in bar) for H2S in water at
 (JEE Lakshya Chemistry M-1) 273K is ___ .
 (JEE Lakshya Chemistry M-1)
INTEGER TYPE QUESTION 57. A CaCl2 aqueous solution at 27°C has an osmotic pressure
49. At 10°C, the osmotic pressure of urea solution is 500 mm 16 atm and density of 1.2 gm/ml. What is depression of
of Hg. The solution is diluted and the temperature is freezing point (in °C) of this solution? [Kf of water = 1.8
raised to 25°C, when the osmotic pressure is found to be K Kg mol–1, R = 0.08 atm litre / mole - K] (multiply final
105.3 mm of Hg. Determine the extent of dilution done answer by 100)
with original solution.  (JEE Lakshya Chemistry M-1)
 (JEE Lakshya Chemistry M-1) 58. Calculate solubility (in moles/litre) of a saturated aqueous
50. Three particles of a solute ‘A’ associate in benzene to solution of Ag3PO4 if the vapour pressure of the solution
form species (A)3. Calculate the freezing point (in °C) of becomes 750 torr at 373 K. (multiply final answer by 54)

Solutions 33
PARIKSHIT (JEE ADVANCED LEVEL)

SINGLE CORRECT TYPE QUESTIONS 5. Calculate solubility of a saturated solution of Ag2CrO4 if


1. A very small amount of non-volatile solute (that does not boiling point of its saturated solution is 373K at a pressure
dissociate) is dissolved is 56.8 cm3 of benzene (density of 760 mm of Hg.
0.889 g cm–3). At room temperature, vapour pressure of this (1 + 1.35 × 10− 5 )
solution is 100 mm Hg while that of benzene is 102 mm Hg. (a) 2.5 × 10–4 M
If the freezing temperature of this solution is 1.3 degree (b) 7.5 × 10–4 M
lower than that of benzene, then what is the value of molal
(c) 5.4 × 10–7 M
freezing point depression constant of benzene?
(d) 1.8 × 10–7 M
(a) 5.07 deg/molal (b) 1.3 deg/molal
 (JEE Lakshya Chemistry M-1)
(c) 3.9 deg/molal (d) 4.97 deg/molal
 (JEE Lakshya Chemistry M-1) MULTIPLE CORRECT TYPE QUESTIONS
2. Using the following information determine the boiling 6. Consider the following vapour pressure composition graph.
point of a mixture containing 1560 gm benzene and 1125 gm Hence
chlorobenzene, when the external pressure is 1000 torr. X
Assume the solution is ideal. Ps
Given: Molar mass of benzene = 78 PA Y V.P.
V.P.
Molar mass of chlorobenzene = 112.5
PB Z
Temperature Vapour pressure Vapour pressure
(0°C) of benzene (torr) of chlorobenzene XAA == 01 XX
A A= 0
X S =1
(torr)
(a) V.P. of A = SZ
80 750 120
(b) V.P. of B = ZY
90 1000 200 (c) V.P. of B = SY
100 1350 300 (d) V.P. of solution at X = SZ + SY
110 1800 400  (JEE Lakshya Chemistry M-1)
120 2200 540 7. The diagram given below represents boiling point
(a) 120 °C (b) 110 °C composition diagram of solution of component A and B,
(c) 100 °C (d) 90 °C which is/are incorrect among the following?
°
TA
 (JEE Lakshya Chemistry M-1)
TB°
3. Pressure over ideal binary liquid mixture containing 10 moles
T (K)
each of liquid A and B is gradually decreased isothermally.
If PA° = 200 mm Hg and PB° = 100 mm Hg, find the pressure
at which half of the liquid is converted into vapour.
(a) 150 mm Hg XB

(b) 166.5 mm Hg (a) The solution shows negative deviation.


(c) 133 mm Hg (b) A-B-interactions are stronger than A-A and B-B.
(d) 141.4 mm Hg (c) The solution is an ideal solution.
(d) The solution shows positive deviation.
 (JEE Lakshya Chemistry M-1)
 (JEE Lakshya Chemistry M-1)
4. The vapour pressure of benzene, toluene and xylene are 80
torr, 20 torr and 10 torr respectively at 25°C. Then which of 8. For the given electrolyte AxBy, the degree of dissociation
the following is possible value of vapour pressure of equimolar ‘α’ of can be given as
binary/ternery solution of these at same temperature? i −1
Assuming all forming an ideal solution with each other. (a) α = (b) i = (1 – α) + xα + yα
( x + y − 1)
110
(a) torr (b) 50 torr 1− i
3 (c) α = (d) None of these
( x − y)
1 −
(c) 45 torr (d) All are possible
 (JEE Lakshya Chemistry M-1)
 (JEE Lakshya Chemistry M-1)

P
34 W JEE (XII) Module-1 CHEMISTRY
9. A liquid mixture having composition corresponding to hand, experimentally obtained values of colligative properties for
point Z in the figure shown is subjected to distillation at associating nature of solute are lower than those obtained theoretically.
constant pressure. The ratio of experimental colligative properties value to theoretical
colligative properties value is called as van't Hoff factor.
12. A weak monoprotic acid (molar mass 180) aqueous
solution of 0.18% w/v at 300 K has observed osmotic
pressure 0.369 atm. What should be its Van't Hoff
factor (i)? (R = 0.082 L-atm/K mole)
(a) 1.2 (b) 1.5
(c) 1 (d) 0.5
Which of the following statement is/are correct about the  (JEE Lakshya Chemistry M-1)
process regarding point Z?
(a) The composition of distillate differs from the mixture. 13. What is observed molar mass (in g/mol) of weak monoprotic
acid in solution in above question?
(b) The boiling point goes on changing.
(c) The mixture has lowest vapour pressure than that for (a) 270 (b) 180
any other composition. (c) 120 (d) 90
(d) Composition of an azeotrope alters on changing the  (JEE Lakshya Chemistry M-1)
external pressure. 14. If equal volume of 0.01 M NaOH is added in the solution of
 (JEE Lakshya Chemistry M-1) above weak acid solution, then what will be new observed
10. Two solutions S1 and S2 containing 0.1 M NaCl(aq.) and 0.08 osmotic pressure at same temperature? Neglect the
M BaCl2(aq.) are separated by a semipermeable membrane. hydrolysis, dissociation of water and any volume contraction
Which among the following statements is/are correct? or expansion. Assume 100% dissociation of the salt formed.
(a) S1 and S2 are isotonic. (a) 0.246 atm (b) 0.369 atm
(b) S1 is hypertonic and S2 is hypotonic. (c) 0.123 atm (d) 0.492 atm
(c) S1 is hypotonic and S2 is hypertonic.  (JEE Lakshya Chemistry M-1)
(d) Osmosis will take place from S1 to S2.
Comprehension (Q. 15 to 16): The composition of vapour over
 (JEE Lakshya Chemistry M-1)
a binary ideal solution is determined by the composition of the
11. 1 mole of a non-volatile solid is dissolved in 200 moles liquid. XA and YA are the mole fractions of ‘A’ in the liquid and
of water. The solution is taken to a temperature TK vapour at equilibrium, respectively.
(lower than the freezing point of solution) to cause ice
formation. After removal of ice, the remaining solution is 15. The value of XA for which (YA – XA) is maximum is
taken to 373 K where vapour pressure is observed to be 740
PAo .PBo – PAo PAo .PBo – PBo
mm of Hg. Identify the correct option. (a) (b)
[Given data: Kf (H2O) = 2 K kg mol–1 and normal boiling PAo – PBo PAo – PBo
point of H2O = 373 K.]
PAo .PBo – PBo PAo .PBo – PAo
(a) 163 moles of ice will be formed. (c) (d)
2000 PAo PBo
(b) F.P. of final solution = 273 – K.  (JEE Lakshya Chemistry M-1)
37 × 18
(c) Freezing point of original solution should be − 10 °C. 16. The vapour pressure of solution at this composition is
18
(d) Relative lowering of vapour pressure of final solution (a) PAo . PBo (b) (PAo – PBo )
1
will be . (c) (PAo + PBo ) (d) 0.5(PAo + PBo )
201
 (JEE Lakshya Chemistry M-1)  (JEE Lakshya Chemistry M-1)
Comprehension (Q. 17 to 19): The experimental values of
COMPREHENSION BASED QUESTIONS colligative properties of many solutes in solution resembles
Comprehension (Q. 12 to 14): Colligative properties i.e., the calculated values of colligative properties.
properties of solution which depend upon the number of particles However in same cases, the experimental value of colligative
present in solution are osmotic pressure, depression in freezing property differ widely than those obtained by calculation. Such
point, elevation in boiling point and lowering in vapour pressure. experimental values of colligative properties are known as
Experimental values of colligative properties for electrolytes Abnormal values of colligative property. Cause for abnormal
are always higher than those obtained theoretically because values of colligative properties are:
electrolytes dissociate to furnish more ions in solution. On the other

Solutions 35
(i) Dissociation of solute: It increases the value of colligative D. 10 mL of 0.1 M HCl s. Freezing point of
properties. e.g.: Dissociation of KCl, NaCl etc. in H2O. aqueous solution is added solution increases
(ii) Association of solute: It decreases the value of colligative to 10 mL of 0.1 M KOH
properties. e.g.: Dimerisation of acetic acid in benzene aqueous solution
17. If degree of dissociation of an electrolyte A2B3 is 25% in a
(a) A→ q, r, s; B→ p, q, r, s; C→ p, q, r; D→ p, r, s
solvent, then
(a) Normal boiling point = Experimental boiling point (b) A→ p, q; B→ q, r, s; C→ p, r, s; D→ p, q, r, s
(b) Normal freezing point < Experimental freezing point (c) A→ p, q, r, s; B→ p, q, r, s; C→ p, q, r, s; D→ p, q, r, s
(c) Normal osmotic pressure = (1/2) Experimental osmotic (d) A→ p, q, r; B→ p, q, s; C→ p, q, r, s; D→ p, q,
pressure  (JEE Lakshya Chemistry M-1)
(d) Normal molecular weight = Experimental molecular
weight 21.
 (JEE Lakshya Chemistry M-1) Column-I Column-II
18. 4 different 100 mL solutions are prepared by mixing 1 gram A. Acetone + CHCl3 p. ∆Smix. > 0
each of NaCl, (NH2)2CO, Na2SO4 and K4[Fe(CN)6] at
B. Ethanol + Water q. ∆Vmix. > 0
temperature T. Correct order of osmotic pressure is
(a) (NH2)2CO solution > NaCl solution > Na2SO4 solution C. C2H5Br + C2H5l r. ∆Hmix. < 0
> K4[Fe(CN)6] solution D. Acetone + Benzene s. Maximum boiling
(b) NaCl solution > Na2SO4 solution > (NH2)2CO solution azeotrope
> K4 [Fe(CN)6] solution t. Minimum boiling
(c) K4[Fe(CN)6] solution > Na2SO4 solution > NaCl azeotrope
solution (NH2)2CO solution
(d) Na2SO4 solution > (NH2)2CO solution > NaCl (a) A→ p ; B→ p, q, t; C→ p; D→ q, t
solution > K4 [Fe(CN)6] solution (b) A→ p, s,; B→ q, t; C→ p; D→ p, q,
 (JEE Lakshya Chemistry M-1) (c) A→ p, r; B→ q, t; C→ p; D→ p, q, t
(d) A→ p, s, r; B→ p, q, t; C→ p; D→ p, q, t
19. One mole l2 (solid) is added in 1 M, 1 litre Kl solution. Then
 (JEE Lakshya Chemistry M-1)
(a) Osmotic pressure of solution increases
(b) Freezing point of solution increases 22.
(c) Relative lowering in vapour pressure decreases Column-I Column-II
(d) No change in boiling point of solution (Solute, degree of (van't Hoff
 (JEE Lakshya Chemistry M-1) ionisation) factor)

A. K2SO4 ; α = 0.85 p. 3.7


MATCH THE COLUMN TYPE QUESTIONS
B. SnCl2 ; α = 0.7 q. 4.0
20. Assuming all the solutes are non-volatile and all solutions
are ideal and neglect the hydrolysis of cation and anion. C. Al(NO3)3 ; α = 0.9 r. 2.7
Column-I Column-II D. Fe2(SO4)3 ; α = 0.75 s. 2.4
A. 10 mL of 0.1 M NaOH p. Osmotic pressure
aqueous solution is mixed of solution (a) A→ r; B→ s; C→ p; D→ q
with 10 mL of 0.1 M HCl decreases (b) A→ p; B→ s; C→ r; D→ q
aqueous solution
(c) A→ s; B→ r; C→ p; D→ q
B. 10 mL of 0.1 M NaOH q. Vapour pressure
(d) A→ q; B→ r; C→ p; D→ s
aqueous solution is added to of solution
10 mL of 0.1 M CH3COOH increases  (JEE Lakshya Chemistry M-1)
aqueous solution
C. 10 mL of 0.1 M HCl r. Boiling point of INTEGER TYPE QUESTIONS
aqueous solution is added solution decreases 23. Equimolar mixture of A and B form an ideal solution
to 10 mL of 0.1 M NH3 at 300 K. The vapour of this solution is condensed
aqueous solution in 2nd container and temperature is maintained at
300 K. The vapour of 2nd container is condensed into 3rd

P
36 W JEE (XII) Module-1 CHEMISTRY
container and temperature is maintained at 300 K. If total NUMERICAL TYPE QUESTIONS
vapour pressure of solution in 3rd container is x mm Hg, (UPTO TWO DECIMAL PLACE)
x
the value of is [P°A, and P°B at 300 K are 100 mm Hg 28. A binary solution of A and B is ideal. If total vapour
9
pressure of this solution is represented as
and 200 mm Hg respectively.]
PS (in mm Hg) = 100 – 50 XB
 (JEE Lakshya Chemistry M-1)
Here, PS ⇒ Total vapour pressure
24. A non-volatile solute P is present in 500 mL solution and XB ⇒ Mole fraction of B in liquid state
molarity is found to be 1 M. A sample of 5 mL of this
This solution is passed through different distillation stages
solution is taken in another container and further 0.4 g of
and at a particular stage, it is found that the mole fraction
P is added and volume is increased by adding water such 4
of A in vapour phase is . The total vapour pressure of
that final volume becomes 10 L. If molarity of diluted 7
solution is 10–3 M, the molar mass of solute P (in g/mol) solution at this stage of distillation is ______ (in mm Hg).
is _________ .
 (JEE Lakshya Chemistry M-1)
 (JEE Lakshya Chemistry M-1)
29. The mole fraction, x of a solute in an ideal saturated
25. A solution comprising 0.1 mol of naphthalene and 0.6 mol
− ∆H  1 1 
of benzene is cooled until some solid benzene freezes out. solution in a solvent is given by log10x = × − 
The solution is then decanted off from solid and warmed to 2.303R  T T0 
353 K, where its vapour pressure is found to be 608 torr. The where DHf and T0 are the molar heat of fusion and the
freezing and normal boiling points of benzene are 278.5 K freezing point of the solute. The melting point of
and 353 K respectively. The amount of benzene that freezes phenanthrene is 100°C and the latent heat of fusion is
out due to cooling is x g. The value of 5x is _________ . 24.3 cal g –1 . Calculate x, (the ideal solubility) of
 (JEE Lakshya Chemistry M-1) phenanthrene (C14H10) in benzene at 25°C.
26. A solution containing compound 'X' in water and a solution  (JEE Lakshya Chemistry M-1)
containing glucose in water were put in a closed system.
30. Mixture of two liquids A and B is placed in cylinder
By doing this, some water vapour was removed from one
containing piston. Piston is pulled out isothermally so that
solution and got condensed in the other. It is found that when
volume of liquid decreases but that of vapour increases.
both the solutions were at equilibrium vapour pressure, one
When negligibly small amount of liquid was remaining,
solution contains 10% (w/w) of 'X' and the other 5% (w/w)
the mole fraction of A in vapour is 0.4. Given PA° = 0.4 atm
glucose. The molar mass (in g/mol) of 'X' is
and P B° = 1.2 atm at the experimental temperature.
 (JEE Lakshya Chemistry M-1) Calculate the total pressure (in atm) at which the liquid
27. Safrole is contained in oil of sassafras and was once used to has almost evaporated. (Assume ideal behaviour).
flavour root beer. A 2.4 mg sample of safrole was dissolved  (JEE Lakshya Chemistry M-1)
in 100.0 mg of diphenyl ether. The solution has a freezing
of 25.64oC. The freezing point of pure diphenyl ether is
26.84oC and the freezing-point-depression constant Kf, is
8.00oC/m. The molecular mass (in g/mol) of safrole is
 (JEE Lakshya Chemistry M-1)

Solutions 37
PYQ'S (PAST YEAR QUESTIONS)

EXPRESSING CONCENTRATIONS [Atomic Masses – Na : 23.0 u, O : 16.0 u, P : 31.0 u]


OF SOLUTIONS  [26 Aug, 2021 (Shift-II)]
 (JEE Lakshya Chemistry M-1)
1. Given below are two statements: one is labelled as Assertion
(A) and the other is labelled as Reason (R). 7. The mole fraction of glucose (C6H12O6) in an aqueous binary
solution is 0.1. The mass percentage of water in it, to the
Assertion (A): 3.1500g of hydrated oxalic acid dissolved in
nearest Integer, is __________ . [3 Sept, 2020 (Shift-I)]
water to make 250.0 mL solution will result in 0.1 M oxalic
acid solution.  (JEE Lakshya Chemistry M-1)
Reason (R): Molar mass of hydrated oxalic acid is 8. Liquids A and B form an ideal solution in the entire
composition range. At 350 K, the vapor pressures of pure
126 g mol–1. [10 April, 2023 (Shift-II)]
A and pure B are 7 × 103 Pa and 12 × 103 Pa, respectively.
In the light of the above statements, choose the correct The composition of the vapour in equilibrium with a solution
answer from the options given below: containing 40 mole percent of A at this temperature is
(a) Both (A) and (R) are true but (R) is NOT the correct  [10 Jan, 2019 (Shift-I)]
explanation of (A).
(a) XA = 0.76 ; XB = 0.24 (b) XA = 0.37 ; XB = 0.63
(b) (A) is false but (R) is true.
(c) XA = 0.28 ; XB = 0.72 (d) XA = 0.4 ; XB = 0.6
(c) (A) is true but (R) is false.  (JEE Lakshya Chemistry M-1)
(d) Both (A) and (R) are true and (R) is the correct 9. The mole fraction of urea in an aqueous urea solution
explanation of (A). containing 900 g of water is 0.05. If the density of the
 (JEE Lakshya Chemistry M-1) solution is 1.2 g cm–3, the molarity of urea solution is _____.
2. The molality of a 10%(V/V) solution of di-bromine solution (Given data: Molar masses of urea and water are 60 g mol–1
in CCl4 (carbon tetrachloride) is 'x'. x = _________ × 10–2 and 18 g mol–1, respectively) [JEE Adv (2019)]
M. (Nearest Integer)  (JEE Lakshya Chemistry M-1)
[Given: molar mass of Br2 = 160 g mol–1
atomic mass of C = 12 g mol–1
atomic mass of Cl = 35.5 g mol–1 SOLUBILITY AND VAPOUR PRESSURE
density of dibromine 3.2 g cm–3 OF LIQUID SOLUTIONS
density of CCl4 = 1.6 g cm–3] [1 Feb, 2023 (Shift-II)] 10. Solid Lead nitrate is dissolved in 1 litre of water. The solution
 (JEE Lakshya Chemistry M-1) was found to boil at 100.15ºC. When 0.2 mol of NaCl is
3. A solution of sugar is obtained by mixing 200 g of its 25% added to the resulting solution, it was observed that the
solution and 500 g of its 40% solution (both by mass). The solution froze at –0.8ºC. The solubility product of PbCl2
mass percentage of the resulting sugar solution is. (Nearest formed is _____________ ×10–6 at 298 K. (Nearest Integer)
Integer) [11 April, 2023 (Shift-I)] Given: Kb = 0.5 K kg mol–1 and Kf = 1.8 kg mol–1. Assume
 (JEE Lakshya Chemistry M-1) molality to be equal to molarity in all cases.
4. When 800 mL of 0.5 M nitric acid is heated in a beaker, [29 Jan, 2023 (Shift-I)]
its volume is reduced to half and 11.5 g of nitric acid is
 (JEE Lakshya Chemistry M-1)
evaporated. The molarity of the remaining nitric acid
solution is x × 10–2M. (Nearest Integer) 11. If O2 gas is bubbled through water at 303 K, the number
of millimoles of O2 gas that dissolve in 1 litre of water is
(Molar mass of nitric acid is 63 g mol–1)
__________. (Nearest Integer)
[26 July, 2022 (Shift-I)]
(Given: Henry’s Law constant for O2 at 303 K is 46.82 k bar
 (JEE Lakshya Chemistry M-1) and partial pressure of O2 = 0.920 bar)
5. The molarity of the solution prepared by dissolving 6.3 g of (Assume solubility of O 2 in water is too small, nearly
oxalic acid (H2C2O4 . 2H2O) in 250 mL of water in mol L–1 negligible) [29 July, 2022 (Shift-II)]
is x × 10–2. The value of x is ______. (Nearest Integer)
 (JEE Lakshya Chemistry M-1)
[Atomic mass : H : 1.0, C : 12.0, O : 16.0]
12. The oxygen dissolved in water exerts a partial pressure of
 [31Aug, 2021 (Shift-I)] 20 kPa in the vapour above water. The molar solubility of
 (JEE Lakshya Chemistry M-1) oxygen in water is ____ × 10–5 mol dm–3.
6. 100 mL of Na3PO4 solution contains 3.45 g of sodium. (Round off to the Nearest Integer).
The molarity of the solution is ____ × 10–2 mol L–1, (Nearest [Given: Henry’s law constant = KH = 8.0 × 104 kPa for O2
Integer)
Density of water with dissolved oxygen = 1.0 kg dm–³]

P
38 W JEE (XII) Module-1 CHEMISTRY
 [17 March, 2021 (Shift-I)] Choose the correct answer from the options given below :
 (JEE Lakshya Chemistry M-1)  [31 Jan, 2023 (Shift-II)]
13. At 35°C, the vapour pressure of CS2 is 512 mm of Hg (a) (B), (D), and (E) only (b) (A), (B), and (D) only
and that of acetone is 344 mm of Hg. A solution of CS2 in (c) (A) and (C) only (d) (B) and (D) only
acetone has a total vapour pressure of 600 mm of Hg. The (JEE Lakshya Chemistry M-1)
false statement among the following is 18. What weight of glucose must be dissolved in 100 g of water
(a) CS2 and acetone are less attracted to each other than to to lower the vapour pressure by 0.20 mm Hg?
themselves (Assume dilute solution is being formed)
(b) Heat must be absorbed in order to produce the solution Given: Vapour pressure of pure water is 54.2 mm Hg at
at 35°C room temperature. Molar mass of glucose is 180 g mol–1.
(c) Raoult's law is not obeyed by this system [11 April, 2023 (Shift-II)]
(d) a mixture of 100 mL CS2 and 100 mL acetone has a (a) 4.69 g (b) 3.59 g
volume of < 200 mL [7 Jan, 2020 (Shift-I)] (c) 2.59 g (d) 3.69 g
 (JEE Lakshya Chemistry M-1)  (JEE Lakshya Chemistry M-1)
14. Liquids A and B form an ideal solution for all compositions 19. A solution containing 2g of a non-volatile solute in 20g of
of A and B at 25°C. Two such solutions with 0.25 and 0.50 water boils at 373.52 K. The molecular mass of the solute
mole fractions of A have the total vapor pressures of 0.3 is gmol–1. (Nearest Integer)
and 0.4 bar, respectively. What is the vapor pressure of pure Given, water boils at 373 K, Kb for water = 0.52 K kg mol–1
liquid B in bar? [JEE Adv 2020] [30 Jan, 2023 (Shift-I)]
 (JEE Lakshya Chemistry M-1)  (JEE Lakshya Chemistry M-1)
15. At room temperature, a dilute solution of urea is prepared 20. When a certain amount of solid A is dissolved in 100 g of
by dissolving 0.60 g of urea in 360 g of water. If the vapour water at 25°C to make a dilute solution, the vapour pressure
pressure of pure water at this temperature is 35 mm Hg, of the solution is reduced to one-half of that of pure water.
lowering of vapour pressure will be: (molar mass of urea = The vapour pressure of pure water is 23.76 mmHg. The
60 g mol–1) [10 April, 2019 (Shift-I)] number of moles of solute A added is________. (Nearest
(a) 0.027 mmHg (b) 0.031 mmHg Integer) [27 July, 2022 (Shift-II)]
(c) 0.028 mmHg (d) 0.017 mmHg  (JEE Lakshya Chemistry M-1)
 (JEE Lakshya Chemistry M-1) 21. The osmotic pressure exerted by a solution prepared by
dissolving 2.0 g of protein of molar mass 60 kg mol–1 in
IDEAL AND NON-IDEAL SOLUTIONS 200 mL of water at 27°C is _____Pa. (nearest Integer)
16. Two open beakers one containing a solvent and the other (Use R = 0.083 L bar mol–1 K–1) [26 June, 2022 (Shift-II)]
containing a mixture of that solvent with a non-volatile
 (JEE Lakshya Chemistry M-1)
solute are together sealed in a container. Over time.
22. Which one of the following 0.06 M aqueous solutions has
(a) The volume of the solution and the solvent does not
change. lowest freezing point? [22 July, 2021 (Shift-II)]
(b) The volume of the solution increases and the volume (a) Kl (b) Al2(SO4)3
of the solvent decreases. (c) C6H12O6 (d) K2SO4
(c) The volume of the solution does not change and the  (JEE Lakshya Chemistry M-1)
volume of the solvent decreases. 23. 40 g of glucose (Molar mass = 180) is mixed with
(d) The volume of the solution decreases and the volume 200 mL of water. The freezing point of solution is _____ K.
of the solvent increases. [7 Jan, 2020 (Shift-II)]
(Nearest Integer)
 (JEE Lakshya Chemistry M-1)
[Given: Kf =1.86 K kg mol–1; Density of water = 1.00 g
COLLIGATIVE PROPERTIES AND cm–3; Freezing point of water = 273.15 K]
ABNORMAL MOLAR MASS  [27 Aug, 2021 (Shift-II)]
17. Evaluate the following statements for their correctness.  (JEE Lakshya Chemistry M-1)
(A) The elevation in boiling point temperature of water Question stem for Question No. (24-25)
will be same for 0.1 M NaCl and 0.1 M urea. The boiling point of water in a 0.1 molal silver nitrate solution
(B) Azeotropic mixtures boil without change in their composition (solution A) is x°C. To this solution A, an equal volume of 0.1
(C) Osmosis always takes place from hypertonic to molal aqueous barium chloride solution is added to make a new
hypotonic solution solution B. The difference in the boiling points of water in the two
(D) The density of 32% H2SO 4 solution having molarity solutions A and B is y × 10–2 °C.
4.09 M is approximately 1.26 g mL−1 (Assume: Densities of the solutions A and B are the same as that
(E) A negatively charged sol is obtained when KI solution of water and the soluble salts dissociate completely.
is added to silver nitrate solution. Use: Molal elevation constant (Ebullioscopic constant).

Solutions 39
Kb = 0.5 K kg mol–1; Boiling point of pure water as 100°C.) 28. The osmotic pressure of a solution of NaCl is 0.10 atm
and that of a glucose solution is 0.20 atm. The osmotic
 (JEE Lakshya Chemistry M-1)
pressure of a solution formed by mixing 1 L of the sodium
24. The value of x is ______. [JEE Adv 2021] chloride solution with 2 L of the glucose solution is
 (JEE Lakshya Chemistry M-1) x × 10–3 atm. x is____. (nearest Integer)
 [4 Sept, 2020 (Shift-II)]
25. The value of |y| is ______. [JEE Adv 2021]
 (JEE Lakshya Chemistry M-1)
 (JEE Lakshya Chemistry M-1) 29. The freezing point of diluted milk sample is found to be
26. On dissolving 0.5 g of a non-volatile non-ionic solute to 0.2°C, while it should have been –0.5°C for pure milk. How
much water has been added to pure milk to make the diluted
39 g of benzene, vapor pressure decreases from 650 mm Hg
sample? [11 Jan, 2019 (Shift-I)]
to 640 mm Hg. The depression of freezing point of benzene
(a) 1 cup of water to 2 cups of pure milk
(in K) upon addition of the solute is ______.
(b) 3 cups of water to 2 cups of pure milk
(Given data: Molar mass and the molal freezing point
(c) 1 cup of water to 3 cups of pure milk
depression constant of benzene are 78 g mol–1 and
5.12 K kg mol–1, respectively) [JEE Adv (2019)] (d) 2 cups of water to 3 cups of pure milk
(JEE Lakshya Chemistry M-1)  (JEE Lakshya Chemistry M-1)
27. A set of solutions is prepared using 180 g of water as a 30. On dissolving 0.5 g of a non-volatile non-ionic solute to
solvent and 10 g of different non-volatile solutes A, B and C. 39 g of benzene, vapor pressure decreases from 650 mm Hg
The relative lowering of vapour pressure in the presence to 640 mm Hg. The depression of freezing point of benzene
of these solutes are in the order [Given, molar mass of (in K) upon addition of the solute is ______.
A = 100 g mol–1; B = 200 g mol–1; C = 10,000 g mol–1] (Given data: Molar mass and the molal freezing point
 [6 Sept, 2020 (Shift-II)] depression constant of benzene are 78 g mol–1 and
(a) A > C > B (b) C > B >A 5.12 K kg mol–1, respectively) [JEE Adv 2019]
(c) A > B > C (d) B > C >A  (JEE Lakshya Chemistry M-1)
 (JEE Lakshya Chemistry M-1)

PW CHALLENGERS
NUMERICAL TYPE QUESTIONS I2(aq) + I–(aq)_______ 
 _______I3–(aq)
(ANSWER UPTO ONE DECIMAL PLACE) 0.1 M KI solution dissolves 12.5 g/L of I2.
(a) Calculate the equilibrium constant K C for the above
1. The molecular weight of a newly synthesized organic equilibrium.
compound was determined by the method of isothermal
(b) Also calculate the freezing point (in K) of the resulting
distillation. In this procedure two solutions, each in an open solution. Assume molarity to be equal to molality and also
calibrated vial, are placed side by side in a closed chamber. assume that conc. of I2 in all saturated solution is same.
One of the solutions contained 9.3 mg of the new compound,  [Kf for water = 1.86 K kg mol– 1]
the other 13.2 mg of azobenzene (MW = 182). Both were  (JEE Lakshya Chemistry M-1)
dissolved in portions of the same solvent. During a period
of 3 days of equilibration, solvent distilled from one vial NUMERICAL TYPE QUESTIONS
into the other until the same partial pressure of solvent was
3. Two solutions of non-volatile solutes A and B are prepared.
reached in the two vials. At this point the distillation of
solvent stopped. Neither of the solutes distilled at all. The The molar mass ratio, M A = 1 . Both are prepared as 5%
MB 3
volume of the two solutions at equilibrium were then read solutions by weight in water. Calculate the ratio of the
on the calibration marks of the vials. The solution containing (∆Tf ) A
the new compound occupied 1.72 mL and the azobenzene freezing point depressions, of the solutions. If the
(∆Tf ) B
solution occupied 1.02 mL. What is the molecular weight two solutions are mixed to prepare two new solution S1 and
(in g/mol) of the new compound? The mass of solvent in S2, the mixing ratio being 2 : 3 and 3 : 2 by volume for S1
solution may be assumed to be proportional to the volume (∆Tf )S1
and S2 respectively what would be the value of ?
of the solution. (∆Tf )S2
2. The freezing point of an aqueous saturated solution of I2 is  (Answer upto two decimal place)
–0.0024°C. More than this can dissolve in an KI solution
because of the following equilibrium:  (JEE Lakshya Chemistry M-1)

P
40 W JEE (XII) Module-1 CHEMISTRY
4. (a) The solubility of iodine per unit volume is 200 times greater of liquid in the right compartment is 74.5 mm in excess of
in ether than in water at a particular temperature. If an that in the left compartment. The temperature of the system
aqueous solution of iodine, 30 mL in volume and containing is maintained at 298K. The number of millimoles in 320 g
2.0 mg of iodine, is shaken with 30 mL of ether and the of haemoglobin is (Given: The density of final solution is
ether is allowed to separate, what quantity of iodine (in mg) 1.013 g/ml, g = 10 m/s2, R = 0.08 L-atm / K-mol)
remains in the water layer?
 (Answer upto three decimal place) SINGLE CHOICE TYPE QUESTIONS
(b) What quantity of iodine (in mg) remains in the water layer  (JEE Lakshya Chemistry M-1)
if only 3 mL of ether is used? 9. A dilute solution contains m mol of solute A in 1 kg of
 (Answer upto three decimal place) a solvent with molal elevation constant K b; The solute
(c) How much iodine (in mg) is left in the water layer if the  A2. What is correct
dimerises in solution as 2A 
extraction in (b) is followed by a second extraction, again expression for the equilibrium constant for this dimer
using 3 mL of ether?  (Answer upto four decimal place) formation? (DTb is the elevation in boiling point for the
 (JEE Lakshya Chemistry M-1) given solution)
K b (K b m + ∆Tb ) K b (K b m + ∆Tb )
5. (a) A liquid mixture of benzene and toluene is composed of (a) K = (b) K =
1 mol of benzene and 1 mol of toluene. If the pressure over (2∆Tb + K b m) 2 (∆Tb + K b m) 2
the mixture at 300 K is reduced, at what pressure (in mm K b (K b m + ∆Tb ) K b (K b m − ∆Tb )
Hg) does the first vapour form? (c) K = (d) K =
(∆Tb − 2K b m) 2 (2∆Tb − K b m) 2
 (Answer upto two decimal place)
 (JEE Lakshya Chemistry M-1)
(b) What is the mole fraction of benzene in the first trace of vapour
formed. (Answer upto two decimal place) 10. The partial molar volumes of propanone (CH3COCH3)
(c) If the pressure is reduced further, at what pressure and butanone (CH3COCH2CH3 ) in a mixture in which
(in mm Hg) does the last trace of liquid disappear? mole fraction of propanone is 0.4 are 75 cm3 mol–1 and
80 cm3 mol–1. Thus, the total volume of solution of total
 (Answer upto two decimal place)
mass 1 kg, is
(d) What is the mole fraction of Toluene in the last trace of
(a) 1175 cm3 (b) 1000 cm3
liquid? (Answer upto two decimal place)
(c) 1200 cm3 (d) 972 cm3
Given: PT° = 32.05 mm Hg, PB° = 103 mm Hg
 (JEE Lakshya Chemistry M-1)
COMPREHENSION BASED QUESTIONS
Comprehension (for question 11 to 13)10 mole of liquid ‘A’ and
INTEGER TYPE QUESTIONS 20 mole of liquid ‘B’ is mixed in a cylindrical vessel containing a
piston arrangement. Initially a pressure of 2 atm is maintained on the
6. In water at 20°C, the Henry’s law constant for oxygen is
solution. Now, the piston is raised slowly and isothermally. Assume
4.6 × 104 atm, and for nitrogen it is 8.2 × 104 atm, where
ideal behaviour of solution and A and B are completely miscible.
the concentration are expressed as mole fractions. If we
are to prepare 99 mole % pure oxygen from air (successive PA° = 0.6 atm and PB° = 0.9 atm
treatments by partially dissolving in water and then all the  (JEE Lakshya Chemistry M-1)
gas is removed under vacuum the water), how many cycles 11. The pressure below which the evaporation of liquid solution
would be necessary to achieve this result? will start, is
 (JEE Lakshya Chemistry M-1) (a) 0.6 atm (b) 0.8 atm
7. When cells of the skeletal vacuole of a frog wee placed in (c) 0.77 atm (d) 0.9 atm
series of NaCl solution of different concentrations at 6°C, it  (JEE Lakshya Chemistry M-1)
was observed microscopically that they remained unchanged 1
in x% NaCl solution, it shrank in more concentrated solution 12. The pressure at which th of the total amount (by mol) of
3
and swells in more dilute solutions. Water freezes from the liquid solution taken initially, will be present in the vapour
x% salt solution at –0.40°C. If the osmotic pressure of the form, is [Given = 7.55]
cell cytoplasm at 6°C is ‘y’× 0.0821 atm, then the value of (a) 0.283 (b) 0.791
‘y’ is (Kf = 1.86 K mol–1 kg) (c) 0.600 (d) 0.652
 (JEE Lakshya Chemistry M-1)  (JEE Lakshya Chemistry M-1)
8. Consider the following arrangements, in which a solution 13. The minimum pressure for the existence of liquid solution is
containing 20 g of haemoglobin in 1 dm3 of the solution is (a) 0.6 atm (b) 0.8 atm
placed in right compartment and pure water is placed in left (c) 0.77 atm (d) 0.9 atm
compartment, separated by SPM. At equilibrium, the height

Solutions 41
Answer Key

CONCEPT APPLICATION
1. (d) 2. (b) 3. (a) 4. (b) 5. [ PBº = 600 mm Hg , PAº = 400 mm Hg ] 6. [< 80 mL]
7. [65.25 g/mol]
8. vapour pressure of pure water = 20.22 Torr.
Mol. weight of non-volatile solute = 54 g/mol. 9. [0.33] 10. [5.08 K kg mole–1] 11. [π = 8.35 atm]
12. [34.89 g/mol.] 13. [i = 1.0753, Ka = 3.07 × 10–3]
BOARD LEVEL PROBLEMS
1. (a) 2. (b) 3. (c) 4. (d) 5. (d) 6. (b) 7. (d) 8. (c) 9. (a) 10. (a)
11. (d) 12. (a) 13. (d) 14. (a) 25. (I)-(d), (II)-(d), (III)-(a), (IV)-(c)

PRARAMBH (TOPICWISE)
1. (d) 2. (c) 3. (d) 4. (b) 5. (c) 6. (a) 7. (b) 8. (d) 9. (c) 10. (d)
11. (a) 12. (c) 13. (d) 14. (c) 15. (b) 16. (d) 17. (d) 18. (c) 19. (d) 20. (c)
21. (b) 22. (a) 23. (b) 24. (b) 25. (b) 26. (a) 27. (d) 28. (b) 29. (a) 30. (b)
31. (a) 32. (b) 33. (c) 34. (c) 35. (c) 36. (c) 37. (a) 38. (a) 39. (c) 40. (d)
41. (a) 42. (a) 43. (a) 44. (c) 45. (b) 46. (b)

PRABAL (JEE MAIN LEVEL)


1. (c) 2. (b) 3. (c) 4. (b) 5. (b) 6. (c) 7. (a) 8. (c) 9. (a) 10. (b)
11. (b) 12. (a) 13. (d) 14. (a) 15. (a) 16. (b) 17. (d) 18. (c) 19. (d) 20. (b)
21. (b) 22. (b) 23. (d) 24. (c) 25. (c) 26. (a) 27. (a) 28. (a) 29. (d) 30. (c)
31. (a) 32. (a) 33. (a) 34. (b) 35. (d) 36. (a) 37. (c) 38. (d) 39. (c) 40. (d)
41. (d) 42. (c) 43. (d) 44. (a) 45. (c) 46. (d) 47. (c) 48. (a) 49. [5] 50. [5]
51. [10] 52. [30] 53. [8] 54. [93] 55. [112] 56. [278] 57. [102] 58. [10]

PARIKSHIT (JEE ADVANCED LEVEL)


1. (a) 2. (c) 3. (d) 4. (d) 5. (a) 6. (a, c, d) 7. (a, b, c) 8. (a, b, c) 9. (c, d) 10. (c, d)
11. (a, c) 12. (b) 13. (c) 14. (a) 15. (b) 16. (a) 17. (c) 18. (b) 19. (d) 20. (c)
21. (d) 22. (a) 23. [20] 24. [80] 25. [78] 26. [380] 27. [160] 28. [71.42] 29. [0.23] 30. [0.67]

PYQ’s (PAST YEAR QUESTIONS)


1. (d) 2. [139] 3. [36] 4. [54] 5. [20] 6. [50] 7. [47] 8. (c) 9. [2.99] 10. [13]
11. [1] 12. [25] 13. (d) 14. [0.20] 15. (d) 16. (b) 17. (d) 18. (d) 19. [100] 20. [6]
21. [415] 22. (b) 23. [271] 24. [100.1] 25. [2.5] 26. [0.97 to 1.06] 27. (c) 28. [167.00] 29. (b)
30. [1.02]

PW CHALLENGERS
1. [76.0] 2. (a) 707.2 (b) 272.6 3. 0.82 4. (a) 0.010 mg I2, (b) 0.095 mg I2, (c) 0.0045 mg I2
5. (a) 67.52, (b) XB = 0.76, (c) P = 49.01 mm Hg, (d) XT = 0.24, 6. [7] 7. [960] 8. [5] 9. (d)
10. (a) 11. (b) 12. (b) 13. (c)

P
42 W JEE (XII) Module-1 CHEMISTRY

You might also like